PDA

نسخه کامل مشاهده نسخه کامل : اتاق ریاضیات(طرح سؤالات)



صفحه ها : 1 2 3 4 5 6 [7] 8 9 10 11 12 13 14 15 16 17 18 19 20

Maxwell_1989
28-10-2008, 08:58
چرا دعواش میکنی بنده خدا رو میلاد جون؟
جناب سعید سی پی یو داریم رو مساله ت می فکریم...
فعلا

saeed_cpu_full
28-10-2008, 15:20
ممنون داداش
من خودم رشتم ریاضیه میدونم علامت چیه و چی میخونن
منظورم اینه که ...........

ببخشید
آخه به جای علامت عضو بودن علامت یورو رو گذاشته بودم

*-P@Y@M-*
28-10-2008, 16:07
سؤال:

مجموعه ی زیر را با نماد های ریاضی نشان دهید
{ 1 ، 11 ، 111 ، 1111 ، ... }

منظور از نماد های ریاضی یعنی مثلا اینطوری بنویسیم:
{ X | X € Z ، 1<x<6 }


فک کنم این طوری باشه:

file:///C:/Documents%20and%20Settings/Payam/Desktop/untitled2.bmp

*-P@Y@M-*
28-10-2008, 16:29
ببخشید من هنوز تازه واردم از bb-code سر در نمی ارم
منظورم این بود:
ده به توانه n , منهای یک, تقسیم بر نه
که n متعلقه به N

میشه یه کمکی هم تو نوشتنه این جور عددا بکنین

*-P@Y@M-*
28-10-2008, 16:45
9/ 1 -( 2^10)

*-P@Y@M-*
28-10-2008, 16:55
ببخشید من همش اشتباه میزنم

ده به توانه n منهای یک تقسیم بر 9
یعنی تو رابطه ی بالا به جا 2,n بذارید
n متعلقه به N (اعداد طبیعی)

saeed_cpu_full
28-10-2008, 17:09
ببخشید من همش اشتباه میزنم

ده به توانه n منهای یک تقسیم بر 9
یعنی تو رابطه ی بالا به جا 2,n بذارید
n متعلقه به N (اعداد طبیعی)

دوست عزیز اگر میشه عکستون رو از طریق سایت

برای مشاهده محتوا ، لطفا وارد شوید یا ثبت نام کنید
آپلود کنید بعدش از طریق کد این عکس رو توی فروم قرار بدین

------------------------------------------------------------------

در مورد جوابت هم دستت درد نکنه اگه میشه دلیلش رو هم بگو

Aless
28-10-2008, 20:37
ببخشید میشه بگید انتگرال ایکس به توان ایکس چی میشه؟

elendil
29-10-2008, 14:38
دوستان اگه میشه این 2 گزاره رو ثابت کنید با معلومات در حد سوم دبیرستان :D:D:D

1 - اگر f تابع حقیقی باشد به طوریکه f(x+y)=f(x)+f(y) و x>y آنگاه f(x)>=f(y)

2 - ثابت کنید اگر a و ذ دو عدد صحیح باشند که بی مخالف صفر آنگاه r , q به طور منحصر به فرد وجود دارد که r بزرگتر مساوی صفر و کوچکتر از قدرمطلق b

Iron
29-10-2008, 15:28
سوال اول غلطه. اگر
f(x)=-x
f(x+y)=-(x+y)=-x-y=f(x)+f(y)a

که نتیجه ذکر شده بدست نمیاد.

*-P@Y@M-*
29-10-2008, 16:13
دوست عزیز اگر میشه عکستون رو از طریق سایت

برای مشاهده محتوا ، لطفا وارد شوید یا ثبت نام کنیدآپلود کنید بعدش از طریق کد این عکس رو توی فروم قرار بدین

------------------------------------------------------------------

در مورد جوابت هم دستت درد نکنه اگه میشه دلیلش رو هم بگو

دلیلشو نمیدونم یه دفعه به ذهنم رسید دیدم درسته . :31:

در مورد عکس هم رفتم تو tinypicاپلود کردم ولی نتونستم تو اواتار بذارم چی کار کنم؟

elendil
29-10-2008, 16:48
سوال اول غلطه. اگر
f(x)=-x
f(x+y)=-(x+y)=-x-y=f(x)+f(y)a

که نتیجه ذکر شده بدست نمیاد.

ببخشید صورت سوال رو من کامل ننوشته بودم صورتش این بود

3 - گزاره زیر را اثبات یا رد کنید :

اگر f تابع ....

مرسی از جواب :D:D:D:D


پ.ن :

شرمنده الان صورت سوال رو دیدم یه قسمتشو اشتباه تایپ کردم صورتش اینه :
1- اگر f تابع حقیقی باشد به طوریکه f(x+y)=f(x)*f(y) و x>y آنگاه f(x)>=f(y)

Iron
29-10-2008, 19:56
مجددا سلام.
تابع 2 به توان (x-) میتونه مثال نقض برای این مورد باشه.

elendil
29-10-2008, 21:56
مجددا سلام.
تابع 2 به توان (x-) میتونه مثال نقض برای این مورد باشه.

مرسی :D:D:D

دومین سوال رو هم دوستان یه لطفی بکنید نگاهی بندازین :D:D:D:D

kasra_khan2003
29-10-2008, 23:45
ببخشید میشه یکی شیوه علمی کشیدن بیضی رو توضیح بده؟

ما تو یه کارخونه صنعتی هستیم، میخوایم از یه ورق آلومینیومی مربع شکل، بیضی در بیاریم. روشش باید راحت عملی باشه.

ممنون

Iron
30-10-2008, 11:28
سلام

برای اینکار یا باید از شابلون استفاده کنید یا بیضی کش و یا اینکه روی یه کاغذ پرینت بگیرید و روی قطعه مورد نظر بچسبونید.

توی کتابهای نقشه کشی صنعتی هم یه روش تقریبی برای اینکار گفته شده.

kasra_khan2003
30-10-2008, 17:59
سلام

برای اینکار یا باید از شابلون استفاده کنید یا بیضی کش و یا اینکه روی یه کاغذ پرینت بگیرید و روی قطعه مورد نظر بچسبونید.

توی کتابهای نقشه کشی صنعتی هم یه روش تقریبی برای اینکار گفته شده.

اون روش صنعتی رو نمیدونید؟

کلا اگه روشی هست لطف کنید بگید! من تا شنبه بیشتر وفت ندارم:31:

zahedy2006
30-10-2008, 18:43
اون روش صنعتی رو نمیدونید؟


تو اتوكد براي شابلون ساختن ابتدا يه متوازي الاضلاع مي كشند سپس وسط اضلاع را با آرك به هم وصل مي كنند.

شايد درست نگفته باشم

B L A S T E R
31-10-2008, 10:16
بچه ها اینارو تعیین دامنه کنید...ممنون
تو اولی رادیکال رو کل عبارت هست...اون اخرش هم جزء صحیحه
[ برای مشاهده لینک ، لطفا با نام کاربری خود وارد شوید یا ثبت نام کنید ]

soheilsmart
31-10-2008, 11:00
بچه ها اینارو تعیین دامنه کنید...ممنون
تو اولی رادیکال رو کل عبارت هست...اون اخرش هم جزء صحیحه
[ برای مشاهده لینک ، لطفا با نام کاربری خود وارد شوید یا ثبت نام کنید ]



دومین سوال رو هم دوستان یه لطفی بکنید نگاهی بندازین

لظفا یه بار سوالتنو واضح بگید!





برای مشاهده محتوا ، لطفا وارد شوید یا ثبت نام کنید

kasra_khan2003
31-10-2008, 11:32
تو اتوكد براي شابلون ساختن ابتدا يه متوازي الاضلاع مي كشند سپس وسط اضلاع را با آرك به هم وصل مي كنند.

شايد درست نگفته باشم

درست و غلطش رو نمیدونم! اما واضح که اصلا نبود.

من یه روش میدونستم، که نجار ها استاده میکنن، به این صورت هست:

مختصات 3 نقطه را نیاز داریم، دو نقطه کانونی کانونی، و یک نقطه که روی محیط بیضی باشه، اون نقطه آخری هم، اگه طول قطر کوچک و بزرگ رو داشتی باشیم، بدست میاد.


[ برای مشاهده لینک ، لطفا با نام کاربری خود وارد شوید یا ثبت نام کنید ]

zahedy2006
31-10-2008, 12:35
درست و غلطش رو نمیدونم! اما واضح که اصلا نبود.

من یه روش میدونستم، که نجار ها استاده میکنن، به این صورت هست:

مختصات 3 نقطه را نیاز داریم، دو نقطه کانونی کانونی، و یک نقطه که روی محیط بیضی باشه، اون نقطه آخری هم، اگه طول قطر کوچک و بزرگ رو داشتی باشیم، بدست میاد.


روش شما كه خود تعريف بيضي ميشه و اگه بتونيد بكشيد كاملا دقيق است
مجموع فاصله از دو نقطه همواره ثابت است

روشي كه من گفتم: شما در نظر بگير يه مربع داري. حالا وصط ضلع هاش رو به هم با آرك- منحني (Arc) وصل كن ميشه دايره. اگه مستطيل باشه باز هم بيضي ميشه ولي تو اتوكد دوستامون با بيضي مي كشيدند.

mehdi1357
31-10-2008, 13:49
سلام
بوسیله استقرا ثابت کنید یک مجموعه n عضوی 2 بتوان n زیر مجموعه دارد
متشکرم

kasra_khan2003
31-10-2008, 17:06
روش شما كه خود تعريف بيضي ميشه و اگه بتونيد بكشيد كاملا دقيق است
مجموع فاصله از دو نقطه همواره ثابت است

روشي كه من گفتم: شما در نظر بگير يه مربع داري. حالا وصط ضلع هاش رو به هم با آرك- منحني (Arc) وصل كن ميشه دايره. اگه مستطيل باشه باز هم بيضي ميشه ولي تو اتوكد دوستامون با بيضي مي كشيدند.

خوب طول و عرض این مستطیل باید چه نسبتی از دو قطر بیضی باشند؟

در ضمن، آرک رو یه توضیح میدید؟! من رشتم نقشه کشی نیست. این رو فقط شنیدم.

popeye
31-10-2008, 17:29
سلام
دوستان عزیز اگر در مورد این مساله راهنمایی بفرمایید ممنون می شم:11:


اگر کره ی x*2+(y+2)*2+(z-1)*2=1را در راستای بردار(3,4,12) بر صفحه ی xy تصویر کنیم کدام ناحیه از این صفحه پوشانده می شود؟


:40:

Iron
01-11-2008, 11:16
اون روش صنعتی رو نمیدونید؟

کلا اگه روشی هست لطف کنید بگید! من تا شنبه بیشتر وفت ندارم:31:

البته گفته بودم نقشه کشی صنعتی نه روش صنعتی!:31:

صفحات 42 و 43 از کتابیکه در این آدرس هست دو روش تقریبی رو معرفی کرده


برای مشاهده محتوا ، لطفا وارد شوید یا ثبت نام کنید

elendil
01-11-2008, 14:16
لظفا یه بار سوالتنو واضح بگید!


- ثابت کنید اگر a و b دو عدد صحیح باشند که بی مخالف صفر آنگاه r , q به طور منحصر به فرد وجود دارد که r بزرگتر مساوی صفر و کوچکتر از قدرمطلق b


این سوال دومی هستش سهیل جان :D:D

mohsen1472
01-11-2008, 23:54
با سلام خدمت شما
رابطه ی زیر در مجموعه اعداد حقیقی تعریف شده است . ایا این رابطه می تواند ضابطه ی یک تابع باشد ؟ ( با دلیل )
2x + y ^2 = 3y

*-P@Y@M-*
02-11-2008, 07:41
با سلام خدمت شما
رابطه ی زیر در مجموعه اعداد حقیقی تعریف شده است . ایا این رابطه می تواند ضابطه ی یک تابع باشد ؟ ( با دلیل )
2x + y ^2 = 3y


تابع نیست
مثال نقضش اینه:



X=0 → 3y-y^2=0 → y(3-y)=0 → y=0 y=3

*-P@Y@M-*
02-11-2008, 07:54
بچه ها اینارو تعیین دامنه کنید...ممنون

تو اولی رادیکال رو کل عبارت هست...اون اخرش هم جزء صحیحه
[ برای مشاهده لینک ، لطفا با نام کاربری خود وارد شوید یا ثبت نام کنید ]



جواب اولی :


sin2π[x]≥0 → sin2π[x]=0→ sinπ[x]=0→π[x]=kπ (k€ Z) → [x]=k→x€ R-




جواب دومی :


fx=tanx/sinx

1- sinx≠0≠ → x≠ kπ →x≠(2k)π/2

2- tanx → x≠(2k+1)π & x≠ 2kπ +π/2 → x≠kπ + π/2 → x≠(2k+1)π/2
x≠kπ/2 →

soheilsmart
02-11-2008, 09:42
خیر رابطه بالا تابع نیست
زیرا در تایع اگر دو زوج مرتب وجود داشته باشد که مولفه اول انها با هم برابر با با هم برابر باشد مولفه دوم هم باید باهم باید برایر باشد!
در این رابطه دو زوج مرتب
(0,0)
va
(0.3)
وجود دارد که مولفه های اول برابرند ولی مولفه های دوم نه! مثال نقض واین رابطه تابع نیست


- ثابت کنید اگر a و b دو عدد صحیح باشند که بی مخالف صفر آنگاه r , q به طور منحصر به فرد وجود دارد که r بزرگتر مساوی صفر و کوچکتر از قدرمطلق b

فک کنم یه چیزی رو جا انداختید ! نقشa وq چی هست تو این سوال؟فک کنم این قسمت رو ج انداخته باشید!

shitan
02-11-2008, 13:34
سلام
معکوس سکانت و کسکانت چیه؟

Niels Henrik Abel
02-11-2008, 14:00
اگه بر اساس سري مك لورن سينوس رو بنوسيم به راحتي حل ميشه سري مك لورن و تيلور sin در حد همون دبيرستانه و براحتي بدست مياد البته توسط هندسه هم اثبات ميشه كه امكانات محدود تايپيكي مانع اين كار ميشه در هر حال من فقط اثبات هم ارزي رو مي نويسم
در رياضيات ما قادر به انجام هر كار منطقي هستيم و بر اساس اين اصل كه جاي توضيح آن اينجا نيست ميتوان sin x را بر اساس يك چند جمله اي به صورت ax^n+bx^n-1+...+z نوشت كه بازاي هر xدر اين معادله معادل آن sinx خواهد بود بر اين اساس ما فقط اگه بتونيم ضرايب رو بدست بياريم مشكل حل خواهد شد
...sinx =ax^n+bx^n-1خواهد شد با مشتق گيري متداول از طرفين و مقدار دادن 0 به ط ضرايب بدست مي ايد كه از انجا sin x هم ارز با x خواهد شد (به كلمه هم ارز توجه شود)و مسئله حل مي شود

سما2003
02-11-2008, 23:51
سلام

ببخشید چه جوری میشه رادیکال رو با هر فرجه ای بدون ماشین حساب پیدا کرد؟ حالا اگه با هر فرجه ای امکانش نیست، با 2 یا 3 .

ممنونم.

elendil
03-11-2008, 14:16
فک کنم یه چیزی رو جا انداختید ! نقشa وq چی هست تو این سوال؟فک کنم این قسمت رو ج انداخته باشید!

والا متن سوال همین بود معلممون هم گفته بود که منحصر به فرد نباشن رو توی پیش میخونین ما فقط با منحصر به فردش میتونیم ثابت کنیم

mst_ab
04-11-2008, 17:18
دوستان یک سوال ریاضی داشتم امیدوارم بتونید کمکم کنید ، یک معادله دیفرانسیل غیر خطی مرتبه ی دوم و درجه 2! از اساتید خواهش می کنم یکی اینو حل کنه ، نگید هم که : این به دردم نمی خوره یا خسلی سخته که خودم می دونم! فقط اونایی که جواب رو بلدن بگن خواهشمندیم ! سرعتی !


برای مشاهده محتوا ، لطفا وارد شوید یا ثبت نام کنید

Parser
06-11-2008, 06:58
سلام

ببخشید چه جوری میشه رادیکال رو با هر فرجه ای بدون ماشین حساب پیدا کرد؟ حالا اگه با هر فرجه ای امکانش نیست، با 2 یا 3 .

ممنونم.

سلام

يكي از راه هاي جالب، استفاده از روش نيوتون براي ريشه يابي است.
مثلاً ريشه ي تابع زير، ريشه ي سوم دو است، با روش نيوتون با هر دقتي كه بخواهيد مي توان مقدار آن را محاسبه نمود.
y=-2+x^3

dariushiraz
06-11-2008, 12:17
با سلام. من می خواهم بدانم جواب انتگرال Lnx چی می شود؟

همچنین جواب انتگرال Lnu و u تابعی از x است.

ممنون.

m1367m2006
06-11-2008, 13:31
با سلام. من می خواهم بدانم جواب انتگرال Lnx چی می شود؟

همچنین جواب انتگرال Lnu و u تابعی از x است.

ممنون.
با استفاده از انتگرال جز به جز داریم

[ برای مشاهده لینک ، لطفا با نام کاربری خود وارد شوید یا ثبت نام کنید ]
if we set [ برای مشاهده لینک ، لطفا با نام کاربری خود وارد شوید یا ثبت نام کنید ] then [ برای مشاهده لینک ، لطفا با نام کاربری خود وارد شوید یا ثبت نام کنید ] and [ برای مشاهده لینک ، لطفا با نام کاربری خود وارد شوید یا ثبت نام کنید ] then
[ برای مشاهده لینک ، لطفا با نام کاربری خود وارد شوید یا ثبت نام کنید ]

با استفاده از تغییر متغییر و استفاده از بخش بالا قسمت دوم براحتی نتیجه می شود.

noghrefam
06-11-2008, 23:10
سلام من يك سؤال دارم از مبحث هندسه دبيرستان:
فرض: يك خط و دو نقطه A و B خارج از آن
حكم : يك نقطه روي خط بيابيد كه مجموع فواصل دو نقطه Aو B از آن نقطه كمترين مقدار را داشته باشد .
( تو رو خدا زود جوابمو يكي بده ):19:

Iron
07-11-2008, 14:36
سلام من يك سؤال دارم از مبحث هندسه دبيرستان:
فرض: يك خط و دو نقطه A و B خارج از آن
حكم : يك نقطه روي خط بيابيد كه مجموع فواصل دو نقطه Aو B از آن نقطه كمترين مقدار را داشته باشد .
( تو رو خدا زود جوابمو يكي بده ):19:

سلام
فرض می کنیم نقطه مورد نظر P باشه. حالا می خوایم اندازه AP بعلاوه اندازهPB حداقل باشه.
الف) اگر A و B در دو طرف خط باشند، طبیعتا این حالت وقتی اتفاق میافته که P روی خط مستقیم واصل نقاط A , B باشه. (براساس قضیه حمار)
ب) حالا اگر هر دوتا یک طرف خط باشند، نقطه C رو طوری درنظر می گیریم که خط مورد اشاره در فرض مساله عمود منصف پاره خط BC باشه. از اونجاکه فاصله هر نقطه روی عمود منصف از دو سر پاره خط به یک اندازه هست، پس نقطه P باید حداقل مجموعه فاصله رو از نقاط A و C داشته باشه. این نقطه رو میشه از روش اشاره شده در قسمت الف پیدا کرد.:31:

cheeky
07-11-2008, 14:42
با سلام
یه سوال در باره مفهوم انتگرال داشتم.
اگر ما از 2X انتگرال معین بگیریم از 1 تا 2 مساحت زیرش رو به ما میده که میشه 3 و ما این 3 رو از روی تابع X^2 بدست میاریم.

[ برای مشاهده لینک ، لطفا با نام کاربری خود وارد شوید یا ثبت نام کنید ]

حالا سوالم اینه که چرا مساحت زیر 2X از 1 تا2 برابر y تابع X^2 که از 1 تا 2 طی کرده میشه؟اصلا مفهومش رو نمی تونم بفهمم.اگه میشه با یک مثال توضیح بدید.
مرسی

Maxwell_1989
07-11-2008, 17:54
فهمیدنش از روی شکل یه کم دشواره سعی کنید از روشهای دیگه درکش کنید.
با تشکر

noghrefam
07-11-2008, 19:38
والا ما كه نفهميديم چي گفتين .خودتون فهميدين ؟!!!!!!!!!:13:
البته ممنون ولي اگه ميشه بيشتر توضيح بده و اگر ممكنه تصويرش رو هم بذار

Iron
08-11-2008, 01:14
سلام

شما فرض کنید به اندازه deltax از نقطه a در جهت مثبت x حرکت کنید که deltax میل می کنه بسمت صفر. از اونجاکه تغییرات تابع f در این بازه ناچیزه، مساحت زیر نمودار f(x) در این بازه میشه

f(a)*deltax

اگر g(x) تابع نشاندهنده مساحت زیر f(x) باشه پس این تابع در بازه deltax باندازه
f(a)*deltax
افزایش پیدا می کنه. پس شیب اون میشه
f(a)*deltax/deltax=f(a)
از اونجا که شیب x^2 در برابر است با 2*x ، پس مساحت زیر منحنی 2*x میشه x^2 .

cheeky
08-11-2008, 20:55
والا ما كه نفهميديم چي گفتين .خودتون فهميدين ؟!!!!!!!!!:13:
البته ممنون ولي اگه ميشه بيشتر توضيح بده و اگر ممكنه تصويرش رو هم بذار

منظورم اینه که:
شما تابع 2x رو رسم کن.بعد مساحت زیرش رو از x=1 تا x=2 بدست بیار.میشه 3.
حالا این 3 رو چطوری بدست آوردی؟این جوری:

[ برای مشاهده لینک ، لطفا با نام کاربری خود وارد شوید یا ثبت نام کنید ]

در واقع ما مساحت زیر 2x از رو x^2 بدست آوردیم.
حالا من میگم این وسط چه اتفاقی داره میفته؟شکل کاملا واضحه.

[ برای مشاهده لینک ، لطفا با نام کاربری خود وارد شوید یا ثبت نام کنید ]


سلام

شما فرض کنید به اندازه deltax از نقطه a در جهت مثبت x حرکت کنید که deltax میل می کنه بسمت صفر. از اونجاکه تغییرات تابع f در این بازه ناچیزه، مساحت زیر نمودار f(x) در این بازه میشه

f(a)*deltax

اگر g(x) تابع نشاندهنده مساحت زیر f(x) باشه پس این تابع در بازه deltax باندازه
f(a)*deltax
افزایش پیدا می کنه. پس شیب اون میشه
f(a)*deltax/deltax=f(a)
از اونجا که شیب x^2 در برابر است با 2*x ، پس مساحت زیر منحنی 2*x میشه x^2 .

متاسفانه باز هم منظور شما رو نفهمیدم اگه میشه یکم توضیح بدید.شیب f(a)*deltax چرا میشه f(a)*deltax/deltax؟

این انتگرال چقدر مفهومیه.خیلی درکش سخته.لطفا کمک کنید،چون اگه اینو نفهمم لاپلاس و سری فوریه که دیگه هیچی.!

m1367m2006
08-11-2008, 21:26
شما تابع 2x رو رسم کن.بعد مساحت زیرش رو از x=1 تا x=2 بدست بیار.میشه 3.
حالا این 3 رو چطوری بدست آوردی؟
دوستان یه ذره صبر کنید
ببینم شما اصلن تعریف انتگرال را بلدی ؟
اگه اره میشه بگی شاید اونجا مورد داری

Iron
09-11-2008, 08:24
متاسفانه باز هم منظور شما رو نفهمیدم اگه میشه یکم توضیح بدید.شیب f(a)*deltax چرا میشه f(a)*deltax/deltax؟

این انتگرال چقدر مفهومیه.خیلی درکش سخته.لطفا کمک کنید،چون اگه اینو نفهمم لاپلاس و سری فوریه که دیگه هیچی.!

سلام
شیب f(a)*deltax نمیشه f(a)*deltax/deltax بلکه
شیب f(x) در نقطه a میشه f(a)*deltax/deltax. چون وقتی x باندازه deltax افزایش پیدا می کنه، f(x) باندازه f(a)*deltax افزایش پیدا می کنه.
شیب هم میشه نسبت تغییر f(x) به تغییر x وقتی که تغییر x کوچیک باشه.

cheeky
09-11-2008, 13:31
دوستان یه ذره صبر کنید
ببینم شما اصلن تعریف انتگرال را بلدی ؟
اگه اره میشه بگی شاید اونجا مورد داری

والا دقیقا نه.
دبیرستان که بودیم میگفتن انتگرال پاد مشتقه.رفتیم کاردانی استادمون گفت دقیقا پاد مشتق نیست و یه سری مستطیل کشید و یه چیزایی گفت که فقط هم خودش فهمید.حالا که اومدیم کارشناسی دو دستی میکوبیم بر سرمون که بالاخره این چعنی چی؟
اصلا بگذارید یه جور دیگه بپرسم.مگه مشتق x^2 نمیشه 2x ؟خب طبق تعریف،مشتق یعنی تغییرات y/x .
خب مشتق x^2 در x=4 میشه 16.حالا اگه یک خط از مبدا تا این نقطه رسم کنیم شیبش میشه 4.حالا این 2x چه ربطی به x^2 داره؟

شکل:

[ برای مشاهده لینک ، لطفا با نام کاربری خود وارد شوید یا ثبت نام کنید ]

m1367m2006
09-11-2008, 13:50
پس بگو
ببین انتگرال مجموع چنتا المانه که میتونه این المان رو خط .سطح .رویه منحنی.حجم و...باشه
وقتی شما انتگرال dx را حساب می کنب یهنی طول x (اگه معین باشه) راحساب کردی وقتی f(x هم داشته باشی میشه
x در یه y که میشه مساحت حالا اگه همینا انتگرال دوگانه بگیریم میشه حجم.

خب مشتق x^2 در x=4 میشه 16
خسته نباشی مشتق x^2 میشه2x در x=4میشه 8 نه 16

vanish
14-11-2008, 23:39
سلام
استاد برنامه نویسی دانشگاه یه تمرین داده که:
برنامه ای بنویسید که تابع رگرسیون را محاسبه کند (اگه درست گفته باشم!!!)
مشکل اینجاست که من اصلآ نمی دونم رگرسیون چی هستش؟!؟!؟
(اگه تونستین توی برنامه نویسیشم یه دست نوازشی به سر من بکشید )

Amir_link
15-11-2008, 14:49
سلام به همگی

دوستان می خواستم بدونم انتگرال زیر ، جوابش چطوری بدست میاد؟!:42:


[ برای مشاهده لینک ، لطفا با نام کاربری خود وارد شوید یا ثبت نام کنید ]

Amir_link
15-11-2008, 14:53
سلام
استاد برنامه نویسی دانشگاه یه تمرین داده که:
برنامه ای بنویسید که تابع رگرسیون را محاسبه کند (اگه درست گفته باشم!!!)
مشکل اینجاست که من اصلآ نمی دونم رگرسیون چی هستش؟!؟!؟
(اگه تونستین توی برنامه نویسیشم یه دست نوازشی به سر من بکشید )

سلام vanish

راستش این مبحث تو کتاب آمار و احتمال هستش ، استاد ما چون عشق پیچوندن بود این مبحث و چند تا مبحث

دیگه رو اصلا درس نداد به همین خاطر نمی تونم کمک چندانی بهت بکنم تنها کمک من اینه که شما این مبحث رو

از روی یکی از کتاب های آمار بخونید فکر نمی کنم سخت باشه

کتاب آمار و احتمال مقدماتی (جواد بهبودیان) خیلی خوب و ساده و با مثالهای خوب مباحث رو توضیح داده اگر بتونید

از روی این کتاب بخونیدش خیلی زود به نتیجه می رسید

sepehr_x50
16-11-2008, 17:52
سلام.

دوستان 1 سوال دارم خیلی ساده ولی خودم توش شک دارم.

(-3 و 1 ) اجتماعش با (4,6)

جوابش چی میشه؟!

پرانتز ها نشان دهنده ی بازه هستند.

مرسی.

shitan
17-11-2008, 10:38
سلام
کسی تقریب عدد پی را تا 10 رقم اعشار با استفاده از توابع ارکی مثل arcsinx واستفاده از سری تیلور میدونه؟:41::10:

mahsa1469
17-11-2008, 17:56
سلام.

دوستان 1 سوال دارم خیلی ساده ولی خودم توش شک دارم.

(-3 و 1 ) اجتماعش با (4,6)

جوابش چی میشه؟!

پرانتز ها نشان دهنده ی بازه هستند.

مرسی.
(6,-3) اجتماعش با (6,4) می شه خود این عبارت

m1367m2006
17-11-2008, 20:29
(6,-3) اجتماعش با (6,4) می شه خود این عبارت
ما که نفهمیدیم شما چی گفتید
اما به هرحال این جوابه
[ برای مشاهده لینک ، لطفا با نام کاربری خود وارد شوید یا ثبت نام کنید ]

farnaz_bahall
18-11-2008, 19:46
سلام

از دوستان کسی می دونه زوایه بین دو قطر یک مکعب مربع چه قدره؟

d810i
19-11-2008, 19:33
کسی قبلا تو المپیاد واترلو بوده؟
اگه کسی هست میتونه جواب سوالات پایان ترم کتاب تابع رو پست کنه؟
ممنون

mamaloo_shz
19-11-2008, 21:37
سلام به همه
من 1 کمک فوری میخوام !!!
من ششنبه (پس فردا) میان ترم ریاضی 1 دارم
هیچی از انتگرال ها بلد نیستم !
در واقع از ابتداش ...
فقط 1 سری از این چیز های ساده مثل انتگرال 2x میشه 2^x به اضافه c را بلدم ... :دی
میشه 1 کمکی کنید، جزوه ای چیزی بدید ، من بخونم ....
مرسی ....
راستی سر فصل ها ، این هاست برای انتگرال :
انتگرال گیری بوسیله تغییر متغیر - استفاده از عکس مسیر قاعده زنجیره ای
حاصل جمع ریمانی و انتگرال معین
قضیه اساسی
تغییر متغیر در انتگرال معین

Parser
20-11-2008, 07:29
سلام

از دوستان کسی می دونه زوایه بین دو قطر یک مکعب مربع چه قدره؟

سلام
فكر مي كنم ArcCos(1/3) باشه.

farnaz_bahall
20-11-2008, 14:20
سلام
فكر مي كنم ArcCos(1/3) باشه.

ممنون!
خودم هم به این جواب رسیدم ولی شک داشتم.

کاپیتان رضائی
22-11-2008, 08:52
سلام
اگه حدی رو با روش هپیتال حل کنیم و مکرر جواب مبهم باشه چی کار کنیم؟
ویا جواب صفر بیاد چه باید کرد؟

Maxwell_1989
22-11-2008, 19:36
باید این قدر ادامه بدی تا جواب بدست بیاد.اگه صفر بشه خب جواب حد هم 0 میشه.فقط اگه با هوپیتال گیری جواب حد +یا- بینهایت شد نمی تونی از این قضیه(هوپیتال)استفاده کنی(نه اینکه بگی جواب حد هم بینهایت میشه ها).باید از روشهای دیگه حد رو حساب کنی.

sayan_se666
22-11-2008, 20:01
با سلام
راجع به رويه ها سوالي داشتم . در مورد اين كه از يك خم چگونه مي توان استوانه به دست اورد.

linkin_park
22-11-2008, 21:54
سوال :
برای قضیه زیر مثال نقضی بیاورید :
اگر حد g(x) وقتی x به a میل کند b شود و حد f(x) وقتی x به b میل کند c شود . آنگاه حد f(gx) وقتی X به a میل کند برابر c میشود .

من 2 ساعت فکر کردم دو تا تابع طوری پیدا کنم که این رابطه بالا رو نقض کنه اما نشد ! شما میتونید کمکم کنید ؟!

DAVODI
22-11-2008, 21:57
اگرx1 و x2 ریشه های x^2-mx+m^2=0 باشد معادله ی درجه دومی که ریشه هایش x1/x2 و x2/x1 باشد چیست؟

ali_hp
22-11-2008, 22:11
سوال :
برای قضیه زیر مثال نقضی بیاورید :
اگر حد g(x) وقتی x به a میل کند b شود و حد f(x) وقتی x به b میل کند c شود . آنگاه حد f(gx) وقتی X به a میل کند برابر c میشود .

من 2 ساعت فکر کردم دو تا تابع طوری پیدا کنم که این رابطه بالا رو نقض کنه اما نشد ! شما میتونید کمکم کنید ؟!
سلام
مثال نقض:
g را تابع ثابت b بگیرید(تابعی که همواره مقدارش b است).و f را تابعی بگیرید که f(b)=c+1 و برای هر x مخالف b داریم f(x)=c.
به وضوح حد g در a برابر b است و حد f در b برابر c است.و حد f o g در a برابر c+1 است.
برای پیدا کردن مثال نقض برای یک حکم،یک راه خیلی خوب اینه که سعی کنیم اثبات کنیم اون حکمو...!و ببینیم که چرا اون حکم غلطه...
قضیه ی بالا هم اگه شرط پیوستگی f در b رو بهش اضافه کنیم،درسته..

Maxwell_1989
23-11-2008, 09:34
گرx1 و x2 ریشه های x^2-mx+m^2=0 باشد معادله ی درجه دومی که ریشه هایش x1/x2 و x2/x1 باشد چیست؟

سلام!
S,P معادله اول رو پیدا کنید.(S=m,P=m^2)بعد یه معادله با شرایطی که خواستید بنویسید(با همون روش x^2-Sx+P،که S میشه جمع x1/x2 و x2/x1 و P میشه ضربشون.)حالا این اس و پی رو یه جوری برحسب m تبدیل کنید.
امیدوارم متوجه شده باشید.خودم که نشدم!!!

sherlockholmz
23-11-2008, 18:18
اگرx1 و x2 ریشه های x^2-mx+m^2=0 باشد معادله ی درجه دومی که ریشه هایش x1/x2 و x2/x1 باشد چیست؟

باسلام،

[ برای مشاهده لینک ، لطفا با نام کاربری خود وارد شوید یا ثبت نام کنید ]



موفق باشيد.

linkin_park
23-11-2008, 19:52
سلام
مثال نقض:
G را تابع ثابت b بگیرید(تابعی که همواره مقدارش b است).و f را تابعی بگیرید که f(b)=c+1 و برای هر x مخالف b داریم f(x)=c.
به وضوح حد g در a برابر b است و حد f در b برابر c است.و حد f o g در a برابر c+1 است.
برای پیدا کردن مثال نقض برای یک حکم،یک راه خیلی خوب اینه که سعی کنیم اثبات کنیم اون حکمو...!و ببینیم که چرا اون حکم غلطه...
قضیه ی بالا هم اگه شرط پیوستگی f در b رو بهش اضافه کنیم،درسته..


ممنون رفیق خیلی مثال با حالی بود ! راستش من تو فکرم دنبال دو تا تابع بودم که مثل فرض باشن ولی وقتی ترکیبشون میکنی اصلا تو اون نقطه حد نداشته باشن . اگه یه مثال نقض اون مدلی هم باشه خیلی عالی میشه .
(در ضمن اگه لطف کنی بگی این مثالی که گفتی چجوری به ذهنت رسید ممنون میشم ) :11::40:

ali_hp
23-11-2008, 21:08
ممنون رفیق خیلی مثال با حالی بود ! راستش من تو فکرم دنبال دو تا تابع بودم که مثل فرض باشن ولی وقتی ترکیبشون میکنی اصلا تو اون نقطه حد نداشته باشن . اگه یه مثال نقض اون مدلی هم باشه خیلی عالی میشه .
(در ضمن اگه لطف کنی بگی این مثالی که گفتی چجوری به ذهنت رسید ممنون میشم ) :11::40:
g رو تابع ثابت b بگیرید،f برای هر x مخالف b برابر c باشه،و در b هم تعریف نشده باشه!اینجوری fog در اطراف a تعریف نمیشه...پس حد fog هم در a تعریف نمیشه...نمیدونم منظورت همین بود یا نه...ولی اگه میخوای دقیقتر جواب بدم،تعریف حدو بگو!دقیق!
من شرو کردم به اثبات قضیه،و فهمیدم چرا غلطه و چرا نمیشه اثباتش کرد،اینو که فهمیدم مثال نقضشم فهمیدم!

DAVODI
24-11-2008, 15:33
با قدردانی کردن از sherlochkholmz و maxwell خیلی متشکرم.

DAVODI
24-11-2008, 15:48
خیلی ممنونم...متشکرم.... Sherlock و maxwell

M O B I N
24-11-2008, 16:28
سلام اگه مسئله ریاضی داستید که می خواستید جوابش رو با راه حل ببینید به سایت زیر برید.

برای مشاهده محتوا ، لطفا وارد شوید یا ثبت نام کنید

sayan_se666
24-11-2008, 20:53
با سلام
راجع به رويه ها سوالي داشتم . در مورد اين كه از يك خم چگونه مي توان استوانه به دست اورد

لطفا جواب سئوال را اگر كسي مي داند بدهد

mehdi_349
24-11-2008, 21:24
سلام دوستان عزیز

من در انتگرال گیری از این تابع به مشکل برخوردم .اگه کسی قبلا با نوع مسائل برخورد داشته کمک کنه

اگه بشه تا فردا عصر که خیلی ممنون میشم

file:///C:/Documents%20and%20Settings/mehdi/Desktop/a.JPG[ برای مشاهده لینک ، لطفا با نام کاربری خود وارد شوید یا ثبت نام کنید ]

Maxwell_1989
25-11-2008, 02:42
x^2=u قرار بدید.

mehdi_349
25-11-2008, 10:29
سلام
دوست عزيز

همين راه رو هم رفتم جواب نميده .
اگه شما تونسته ايد؛ مراحل كار رو با تشريح برام بنويسيد

ممنون

shahin.khoobehi
28-11-2008, 09:19
سلام دوستان یه سوال استدلال داشتم.....
100 زندانی داریم که انها میتوانند برنامه ای با هم بریزند.انها را به صف کرده و روی سر هر کدام از انها یک کلاه قرمز یا ابی میگذاریم.هیچکس نمیتواند رنگ کلاه خود را تشخیص دهد و هر فرد رنگ کلاه افراد جلوی خود را میتواند ببیند(نفر اول برای نفر صدم را هم میتونه ببینه) با اغاز از نفر صدم(نفر صدم عقبتر از همه است) از هر نفر به ترتیب رنگ کلاه او را میپرسیم کسی که رنگ کلاهش را نادرست اعلام کند....اعدام خواهد شد!حداکثر چند نفر میتوانند به طور حتم زنده بماند.؟


و اما چند نکته:
1)زندانی ها فقط حق دارند بگویند قرمز یا ابی.
2)نمیتوانند تلفظ را تغییر بدهند
3)صدای فرد را همه میتوانند بشنوند.
3)مثلا زندانی ها با هم قرار میگذارن که:هرکی رنگ کلاه جلویی خود را بگوید.(یعنی اول برای دوم سوم برای چهارم و.....اونوقت نفر دوم که صدای اولی رو شنیده و میدونه که اون رنگ کلاه او را گفته بنابراین زنده میماند.)به این ترتیب 50 نفر زنده می ماند.


می خوام کاری کنید که بیشتر زنده بمانند.فک کنم 99 نفر هم بشه....

mohsen1472
29-11-2008, 23:54
با سلام
فکر کنم این سوال من ربطی به مطالب گفته شده نداشته باشد اما لطفا جواب دهید و کمکم کنید
من در دبیرستان تیزهوشان درس می خوانم
لطفا کتاب هایی برای حسابان و جبر و احتمال معرفی کنید که سطحش بالا بوده و مخصوص مدارس خاص می باشد.
خدانگهدار

Maxwell_1989
30-11-2008, 08:41
کانون بنفشا فک کنم خوب باشه.

ali_hp
30-11-2008, 11:38
با سلام
فکر کنم این سوال من ربطی به مطالب گفته شده نداشته باشد اما لطفا جواب دهید و کمکم کنید
من در دبیرستان تیزهوشان درس می خوانم
لطفا کتاب هایی برای حسابان و جبر و احتمال معرفی کنید که سطحش بالا بوده و مخصوص مدارس خاص می باشد.
خدانگهدار
انتشارات خوشخوان و مبتکران کتابهای مخصوص استعدادهای درخشان دارن،که کتابای خیلی خوبی هم هستن...

minoo.math
30-11-2008, 11:41
بسم الله الرحمن الرحیم

با سلام خدمت دوستان خوب p30world

احساس قوی بنده - به عنوان یک معلم کوچک ریاضی- این است که در این سایت جای یک اتاق بحث و گفتگو پیرامون ریاضیات خالی است. فکر می کنم سرفصل مطالب مهم باید اینها باشد:

1. طرح و حل مسائل مختلف ریاضی در هر سطحی که باشد - دبیرستانی و دانشگاهی.

2. آموزش دروسی که معمولا دانش آموزان و دانشجویان به آن نیاز دارند.

3. ارائه نمونه سوالات امتحانی برای استفاده دانش آموزان و دانشجویان. (هر چند که تایپیکی در این زمینه در p30world
وجود دارد).

4. معرفی سایتهای خوب ریاضی- فارسی و غیر فارسی.

5. رفع اشکال دانش آموزان و دانشجویان و جواب به سوالات آنها.

6. ارائه تستهای کنکور کارشناسی و کارشناسی ارشد ریاضی برای کسانی که به این تستها دسترسی لازم ندارند.

7. معرفی و آموزش نرم افزارهای ریاضی مانند Maple- Mathematica- Matlab -...

8. مقالات خوب ریاضی در سطوح مختلف.

9. بحث پیرامون مسائل آموزشی کشور و جهان و بررسی نقاط ضعف قوت آن.

10. ...

================================================== =======

برای شروع، حل این مساله را به بحث می گذاریم:

با استفاده از اطلاعات معمولی ریاضی - در حد درس حسابان سوم ریاضی - ثابت کنید که


[ برای مشاهده لینک ، لطفا با نام کاربری خود وارد شوید یا ثبت نام کنید ]

(هدف ارائه راه حل ساده تری غیر از روش ارائه شده در حسابان است. در ضمن نمی خواهیم از قاعده هوپیتال استفاده کنیم.)

================================================== =======

منتظر مطالب دوستان هستیم

ارسال متن: 30 فروردین 1385
سلام
لطفا مرا در حل مسائل زیر یاری کنید

1- اگر شعاع دایره ای 100% افزایش یابد ، مساحت آن چقدر افزایش می یابد؟
2- اگر عقربه ساعت شمار یک ساعت 18 درجه طی کند ، عقربه دقیقه شمار چقدر؟
3- اگر شعاع دایره ای 10% کم شود محیط آن چقدر کم میشود؟
4- کالایی 20% کاهش می یابد برای آنکه دوباره به قیمت اولیه برگردد چند درصد باید افزایش یابد؟
5- شیر a در 1 ساعت شیر B در 2ساعت و شیر C در 4ساعت منبعی را پر می کنند اگر 3شیر همزمان باز شوند منبع در چه مدت پر می شود؟
با تشکر

Maxwell_1989
30-11-2008, 19:38
سلام!

جواب 1-وقتی شعاع صد درصد زیاد بشه یعنی دو برابر میشه پس مساحت 4 برابر میشه چون مساحت با مجذور شعاع در ارتباطه. (مساحت=پی آر دو)

جواب2-18 درجه یعنی یک بیستم کل دایره(از نظر درجه).کل یک دور عقربه ساعت شمار 12 ساعته.پس این میشه یک بیستم 12 ساعت یعنی0/6 ساعت یعنی 36 دقیقه.عقربه دقیقه شمار هر 1 ساعت یک دور کامل میزنه.پس حالا 36/60 دور میزنه که میشه 0/6 دور یعنی 0/6*360=216 درجه.

جواب3-محیط دایره اولیه میشه "دو پی آر" و دومی میشه "دو پی در 0/9 آر" که وقتی از هم کم کنیم میشه "دو پی در یکدهم آر".

جواب4-همان 20 درصد(عقلا جواب این میشه-حالا شاید یه نکته انحرافی داره که من نمی گیرم)

جواب5-فرمولش اینه:1 تقسیم بر (یک به آ+یک به بی+یک به سی) جواب میشه 4/7(تقسیم) ساعت.

سوالی بود در خدمتم-ضمنا از آی دی تون تشکر می کنم که مت رو توش به کار بردید.من دیوونه ی ریاضیاتم!

ali_hp
30-11-2008, 20:00
سلام!
جواب4-همان 20 درصد(عقلا جواب این میشه-حالا شاید یه نکته انحرافی داره که من نمی گیرم)

سلام
20 درصد قیمت اولیه باید زیاد شه.(قیمت اولیه=قیمت قبل از کاهش)اما وقتی قیمتو کاهش دادیم،معمولش اینه که در صدمونو بر حسب قیمت کاهش یافته بیان کنیم.یعنی قیمت بعد از کاهش ما که 80 درصد قیمت اولیه هست،باید یک چهارمشو(یا 25 درصدشو) بهش اضافه کنیم تا بشه همون قیمت اولیمون،یعنی جواب میشه 25 درصد.

Maxwell_1989
30-11-2008, 20:46
آهان.مرسی.

mohsen1472
01-12-2008, 14:16
با سلام
در رابطه با کتاب های پیشرفته حسابان و جبر و احتمال اگر نام کتاب ها و نویسندگان آن ها را بگوییید ممنون می شوم.

mm00
01-12-2008, 17:25
میشه این از این موضوعات هر کدوم 1 سوال واسم پیدا کنید
حجم حاصل از دورانمحصور بین 2 منحنی و محور x و Y ها 2تا میشه این
طول قوس منحنی
قضیه لایت نیدز

Maxwell_1989
01-12-2008, 17:39
سلام!
برای اولی:حجم حاصل از دوران ناحیه ی محصور بین توابع sinx و cosx از 0 تا پی دوم=؟
برای دومی:طول قوس منحنی y=رادیکالx چه مقداری است؟
در مورد سومی:منظورتون قضیه ی لایب نیتز نیست؟اگه هست کدوم قضیه ش؟(ماشاءالله این فیلسوف و ریاضیدان بزرگ چندین قضیه به نام خودش داره)

rouhallah
02-12-2008, 12:56
بنام خدا
با سلام
ببخشید اگه سوالم پیش پا افتادس
میخواستم بپرسم که در هذلولی متساوی الساقین که در آن a=b هستش چطور میشه فهمید این هذلولی قائم هست یا افقی که بعد از اون بتونیم کانونها و راسها رو پیدا کنیم
با تشکر

Maxwell_1989
02-12-2008, 19:12
باید ببینی ضریب منفی کجاس اگه پشت x^2 بود قائمه و بالنقض.
البته فک کنم!

Parser
04-12-2008, 06:59
بنام خدا
با سلام

میخواستم بپرسم که در هذلولی متساوی الساقین که در آن a=b هستش چطور میشه فهمید این هذلولی قائم هست یا افقی که بعد از اون بتونیم کانونها و راسها رو پیدا کنیم
با تشکر

سلام
اوّل بايد ديد كه آيا در معادله ي اين هذلولي جمله اي به صورت bxy وجود دارد يا نه. (b ضريب xy است)
اگر چنين جمله اي نبود (يا به عبارتي ديگر b=0) هذلولي مايل نيست. يا افقي است يا قائم.
اكنون اگر ضريب x^2 مثبت بود وضريب y^2 منفي، هذلولي افقي است.
و اگر ضريب x^2 منفي بود وضريب y^2 مثبت، هذلولي قائم است.
البتِه بايد توجّه داشت كه اگر مركز هذلولي در معادله ي آن صدق كرد، اين معادله مربوط به دو خطّ متقاطع است.

shiraz_670
04-12-2008, 14:04
هر سوال ریاضی و فیزیک داشتید اینجا بزارید

برای مشاهده محتوا ، لطفا وارد شوید یا ثبت نام کنید

shiraz_670
04-12-2008, 14:07
سلام آقاي مفيدي . خسته نباشيد . كار بسيار خوبي هست
من هم باهاتون موافقم
جزوه ریاضی 2 فصل اول
[ برای مشاهده لینک ، لطفا با نام کاربری خود وارد شوید یا ثبت نام کنید ]

ub6ib9vb69
06-12-2008, 14:00
سلام خدمت دوستان عزیز...
من یه سوال دارم خواهش میکنم یکی کمک کنه.
سوال:خرگوشی از مبدا مختصات دکارتی در جهت مثبت محور y ها و با تندی a)a شتاب) شروع به دویدن میکند,همزمان با آن روباهی از نقطه ی A به مختصات (0.c) با تندی b به تعغیب خرگوش میپردازد,مسیر روباه را مشخص کنید؟
با تشکر...

math_m
07-12-2008, 11:45
انتگرال ناسره ( e^(-x^2 ∫ از 0 نا بينهايت ؟؟؟؟//

math_m
07-12-2008, 12:01
با سلام
كسي جواب انتگرال ناسره e^(-x^2)dx∫ (از 0 تا بينهايت) را بلد هست؟؟؟؟؟؟؟؟؟؟؟؟؟؟؟//

math_m
07-12-2008, 12:27
با انتگرال جزء به جزء اين طوري ميشه :پ
∫(from0 to x)e^(-x^2)dx= -1/2∫1/x d(-e^(-x^2))
= -1/2[e^(-x^2)] -1/2∫e^(-x^2)dx/x^2
= e^(-x^2)/2x + 1/4∫d(e-x^2)/x^3
= e^(-x^2)/2x - e^(-x^2)/4x^3...

بنابراين erf(x) = 1- ( e^(-x^2)/sqr(P) ) (1-1/2x^3)
=1/2sqr(P)erf(x)
جواب :
= 1/2 sqr(P). 1/2e^(-x^2) / (x+1/ (2x+2/(x+3/(2x+4/(x+...
ولي اين از صفر نا بينهايت چي ميشه ؟

Maxwell_1989
07-12-2008, 19:15
میشه پی چهارم.

mohsen1472
08-12-2008, 20:32
با سلام
لطفا ثابت کنید اگر نیمسازهای دو زاویه در یک مثلث برابر باشند آن مثلث متساوی الساقین است.

saeed-d
08-12-2008, 21:59
[ برای مشاهده لینک ، لطفا با نام کاربری خود وارد شوید یا ثبت نام کنید ]
در این شکل چهار ضلعی bcde مربع است و مثلث abc متساوی الاضلاع است.
اگر ضلع مربع 1 واحد باشد شعای دایره چند واحد است.(اگه بگید راه حل نداره یا غلط دروغ گقتین چون 7-8 نفر این سوال رو حل کردن)
لطفا با راه حل تو ضیح دهید

mm00
09-12-2008, 16:13
2 مثال از قضیه تسلط
1 مثال تیلور
1مثال مکلورن میخوام
مربوط به انتگرال

1731
09-12-2008, 16:54
[ برای مشاهده لینک ، لطفا با نام کاربری خود وارد شوید یا ثبت نام کنید ]
در این شکل چهار ضلعی bcde مربع است و مثلث abc متساوی الاضلاع است.
اگر ضلع مربع 1 واحد باشد شعای دایره چند واحد است.(اگه بگید راه حل نداره یا غلط دروغ گقتین چون 7-8 نفر این سوال رو حل کردن)
لطفا با راه حل تو ضیح دهید

طبق قضیه در هندسه 2 داریم شعاعی که بر خط de عمود شود آن را نصف می کند
از طرفی اگر شعاع را تا بالا ادامه دهیم از وسط bc می گذرد و به علت نوع مثلث ارتفاع بر میانه منطبق است پس امتداد شعاع به a می رسد بعد با فیثاغورث اندازه ی قسمتی از شعاع که در مثلث است حساب می شود و با 0.5 جمع می شود.
1.3تقریبا

کاپیتان رضائی
09-12-2008, 21:43
اگه بگید راه حل نداره یا غلط دروغ گقتین چون 7-8 نفر این سوال رو حل کردن
من جوابشو میدونم اما نمیگم چون هشت نفر جواب دادن دیگه
یه مساله بگو که جدید باشه

1731
10-12-2008, 18:35
این حد بسیار زیبایی است
تنها یک راه حل دارد
راهنمایی: جواب صفر نمی شود!

[ برای مشاهده لینک ، لطفا با نام کاربری خود وارد شوید یا ثبت نام کنید ] ([ برای مشاهده لینک ، لطفا با نام کاربری خود وارد شوید یا ثبت نام کنید ])

Maxwell_1989
10-12-2008, 20:03
خودتون سه بار پشت سر هم از قضیه ی هوپیتال استفاده کنید.من حالش رو ندارم!

1731
10-12-2008, 20:37
خودتون سه بار پشت سر هم از قضیه ی هوپیتال استفاده کنید.من حالش رو ندارم!

من نمی دونم هوپیتال چیه!!!
فقط رفع ابهام بلدم!!

کاپیتان رضائی
10-12-2008, 20:51
این حد بسیار زیبایی است
تنها یک راه حل دارد
راهنمایی: جواب صفر نمی شود!

[ برای مشاهده لینک ، لطفا با نام کاربری خود وارد شوید یا ثبت نام کنید ] ([ برای مشاهده لینک ، لطفا با نام کاربری خود وارد شوید یا ثبت نام کنید ])
با اجازه داش ماکس ول :
میتونیم از هم ارزی تانژانتی استفاده کنیم
صورت رو از هم ارزی و اگه شد مخرجو هم از هوپیتال

Iron
10-12-2008, 21:51
[ برای مشاهده لینک ، لطفا با نام کاربری خود وارد شوید یا ثبت نام کنید ]
در این شکل چهار ضلعی bcde مربع است و مثلث abc متساوی الاضلاع است.
اگر ضلع مربع 1 واحد باشد شعای دایره چند واحد است.(اگه بگید راه حل نداره یا غلط دروغ گقتین چون 7-8 نفر این سوال رو حل کردن)
لطفا با راه حل تو ضیح دهید

این 0.5 از کجا اومد؟


طبق قضیه در هندسه 2 داریم شعاعی که بر خط de عمود شود آن را نصف می کند
از طرفی اگر شعاع را تا بالا ادامه دهیم از وسط bc می گذرد و به علت نوع مثلث ارتفاع بر میانه منطبق است پس امتداد شعاع به a می رسد بعد با فیثاغورث اندازه ی قسمتی از شعاع که در مثلث است حساب می شود و با 0.5 جمع می شود.
1.3تقریبا

اگر مرکز دایره o باشه و نقطه برخورد امتداد ao با ضلع پایینی مربع f و نقطه g وسط ad باشه که طبیعتا og بر ad عموده،
آنگاه aog با adf متشابهه چون قائم الزاویه اند و یک زاویه مشترک دارند طول ad و af و در نتیجه نسبت اونهابا استفاده از رابطه فیثاغورث قابل محاسبه هست. طول ag نیزنصف ad هست، پس معلومه. با توجه به تشابه مثلثها طول ao بدست میاد.

Maxwell_1989
11-12-2008, 09:49
با اجازه داش ماکس ول :
خواهش می کنم عزیز.

قضیه هوپیتال میگه:اگه از صورت و مخرج یه کسر مشتق بگیریم،حدش با حد اون کسر برابره.پس برای حل حدود صفرصفرم یا بینهایت/بینهایت میشه از صورت و مخرج مشتق گرفت(جدا جدا) و بعد حد اونو بدست اورد.

1731
11-12-2008, 14:23
این 0.5 از کجا اومد؟

به نکته ی جالب اشاره کردی!
نمی دونم!!!

قضیه هوپیتال میگه:اگه از صورت و مخرج یه کسر مشتق بگیریم،حدش با حد اون کسر برابره.پس برای حل حدود صفرصفرم یا بینهایت/بینهایت میشه از صورت و مخرج مشتق گرفت(جدا جدا) و بعد حد اونو بدست اورد.

همان طور که گفتم من این راه ها را بلد نیستم
راستش این سوال را استادمون در کلاس مطرح کرد من هم از روش های عادی حل کردم
استادمون می گفت این سوال فقط همین راه حل را دارد که من نوشتم
برای فکر کردن سوال خوبیه
امتحان کنید اگه از راه هایی که می گید حل نشد حلش را بگم!
جوابش باید 6 در بیاد

کاپیتان رضائی
11-12-2008, 15:14
این حد خیلی راحته
چون معلمتون راههای پیشرفته حلشونگفته ، گفته که فقط یه راه حل داره

کاپیتان رضائی
11-12-2008, 15:23
این حد بسیار زیبایی است
تنها یک راه حل دارد
راهنمایی: جواب صفر نمی شود!

[ برای مشاهده لینک ، لطفا با نام کاربری خود وارد شوید یا ثبت نام کنید ] ([ برای مشاهده لینک ، لطفا با نام کاربری خود وارد شوید یا ثبت نام کنید ])
یه راه ساده هم ارزی اینه که وقتی حد به صفر بره ، از جلوی سینوس و تانژانت میشه عبارت شاملشو برداشت
تو این حد هم میتونی بنویسی
ایکس منهای 2ایکس منهای 3ایکس بر ایکس بتوان سه
حالا اگه صفر رو بذاری جای ایکس، جواب صفر صفرم میاد . پس حالا از مخرج و صورت مشتق بگیری میشه روش هوپیتال
بعد از این کار جوابش میشه منهای 4 بر 3ایکس بتوان 2
در قضیه داریم که وقتی درجه مخرج از صورت بیشتر باشه جواب حد صفر میشه ولی تو میگی که جوابش صفر نمیاد
والا من که دیگه چیزی به ذهنم نمیاد

Maxwell_1989
11-12-2008, 17:35
شما هوپیتال برو.6میشه.

olumpc
12-12-2008, 16:49
salam dooste aziz khob age az hopital ham estefadeh nakonim mitoonim az hamarzi estefade konim . midoonim ke lim sinx =0 vaghti xbe samte 0 meyl mikone .
سلام دوستای گلم می خواستم ببینم ln(x)=x ریشه داره یا نه

P30 Love
12-12-2008, 17:30
سلام خسته نباشيد
اثبات δ<ε دلتا
تو مبحث تعریف حد
با تشكر

کاپیتان رضائی
12-12-2008, 19:29
سلام خسته نباشيد
اثبات δ<ε دلتا
تو مبحث تعریف حد
با تشكر

خب که چی؟
جواب دادی یا سوال کردی؟؟

Maxwell_1989
12-12-2008, 19:34
سلام دوستای گلم می خواستم ببینم ln(x)=x ریشه داره یا نه
سلام-نمودار هر دو رو رسم کن اگه هم رو قطع کردن یعنی جواب داره وگرنه نه!

mahsa1469
13-12-2008, 14:10
سلام دوستای گلم می خواستم ببینم ln(x)=x ریشه داره یا نه
ریشه داره و اگه اشتباه نکنم ریشش می شه 10

کاپیتان رضائی
13-12-2008, 14:13
ریشه داره و اگه اشتباه نکنم ریشش می شه 10
خب دیگه پیداش کردی که!

ali_hp
13-12-2008, 18:11
سلام دوستای گلم می خواستم ببینم ln(x)=x ریشه داره یا نه
سلام
ln برای اعداد مثبت تعریف میشه،پس فرض می کنیم x>0 .اگر x بین صفر و یک باشه،طرف چپ معادله منفی میشه،و طرف راستش مثبت،پس x ها ی بین صفر و یک جواب نیستن.حالا تابع f(x)=x - lnx را در نظر بگیرید،داریم f(1)=1 .همچنین مشتق این تابع برای x های بزرگتر مساوی یک،نا منفی است،پس f برای x های بزرگتر مساوی یک صعودی است.و داریم f(x) >= f(1)=1 پس f ریشه ندارد.و معادله lnx=x جواب ندارد.

ali_hp
13-12-2008, 19:35
این حد بسیار زیبایی است
تنها یک راه حل دارد
راهنمایی: جواب صفر نمی شود!

[ برای مشاهده لینک ، لطفا با نام کاربری خود وارد شوید یا ثبت نام کنید ] ([ برای مشاهده لینک ، لطفا با نام کاربری خود وارد شوید یا ثبت نام کنید ])
سلام
جواب منفی بی نهایت میشه!کافیه یه tgx/x ازش جداکنیم!فکر می کنم اون حدزیبایی که شما می خواستی بگی به جای اولین منها تو صورت کسر بعلاوست،درسته؟

Maxwell_1989
13-12-2008, 23:57
1621 تایید میشه.نمودار رو هم که رسم کنید به همین نتیجه می رسید.ایکس و ال ان ایکس هیچ جا هم رو قطع نمی کنن.

mahsa1469
14-12-2008, 11:18
آره حق با دوستانه جواب می شه تهی

panizir
14-12-2008, 16:15
سلام
من يه سوال مثلثات دارم. به نظر يه كم سخت مي رسه اما مي خواستم يكي برام اثباتش كنه:
8Sin(p/7)*Sin(2p/7)*Sin(3p/7)=√7

ali_hp
14-12-2008, 18:17
سلام
من يه سوال مثلثات دارم. به نظر يه كم سخت مي رسه اما مي خواستم يكي برام اثباتش كنه:
8Sin(p/7)*Sin(2p/7)*Sin(3p/7)=√7
سلام
دقت کنید که 0 و 2p/7 و 4p/7 و6p/7 و 8p/7 و 10p/7 و 12p/7 ریشه های معادله sin7x=0 هستند،همچنین:

[ برای مشاهده لینک ، لطفا با نام کاربری خود وارد شوید یا ثبت نام کنید ]^3x&plus;112\sin^5x-64\sin^7x

بنابر این sin(2p/7),sin(4p/7),sin(6p/7),sin(8p/7),sin(10p/7),sin(12p/7),sin0 ریشه های معادله زیر هستند:

[ برای مشاهده لینک ، لطفا با نام کاربری خود وارد شوید یا ثبت نام کنید ]^3&plus;112x^5-64x^7=0

اما sin0=0

پس sin(2p/7),sin(4p/7),sin(6p/7),sin(8p/7),sin(10p/7),sin(12p/7)l ریشه های معادله زیر هستند:

[ برای مشاهده لینک ، لطفا با نام کاربری خود وارد شوید یا ثبت نام کنید ]^2&plus;112x^4-64x^6=0

طبق قضیه ای می دانیم حاصلضرب ریشه های یک چند جمله ای برابر است با جمله ثابت ان ، تقسیم بر ضریب جمله با بزرگترین درجه،از طرفی:

[ برای مشاهده لینک ، لطفا با نام کاربری خود وارد شوید یا ثبت نام کنید ]{4\pi}{7}=\sin\frac{3\pi}{7}&space;\\ &space;\sin\frac{6\pi}{7}=\sin\frac{\pi}{7}=-\sin\frac{8\pi}{7}\\&space;\\\sin&space;\frac{10\pi}{7}=-\sin\frac{3\pi}{7}&space;\\\\&space;\sin\frac{12\pi}{7}=-\sin\frac{2\pi}{7}

پس داریم:


[ برای مشاهده لینک ، لطفا با نام کاربری خود وارد شوید یا ثبت نام کنید ]{7}{64}=\sin\frac{2\pi}{7}*\sin\frac{4\pi}{7} *\sin\frac{6\pi}{7}*\sin\frac{8\pi}{7}*\sin\frac{1 2\pi}{7}*\sin&space;\frac{12\pi}{7}=\\&space;\\&space;-(\sin\frac{\pi}{7}*\sin\frac{2\pi}{7}*\sin\frac{4\ pi}{7})^2


که حکم را نتیجه می دهد.

panizir
14-12-2008, 19:13
ببخشيد من درست راه حلتونو متوجه نشدم. مي شه يه كم بيشتر توضيح بدين؟

ali_hp
14-12-2008, 21:56
با سلام
لطفا ثابت کنید اگر نیمسازهای دو زاویه در یک مثلث برابر باشند آن مثلث متساوی الساقین است.
من این مساله رو با استفاده از محاسبه طول نیمساز بر حسب اضلاع حل کردم،یکم طولانی میشه ولی راحته!و راحت به ذهن میرسه و عجیب نیست!راه حلی که اینجا میگم نمیدونم مال کیه!ولی بی نهایت قشنگه....
از دو قضیه زیر در حل این مساله استفاده میشه:
قضیه یک:در مثلث مفروض abc داریم b>c اگر و تنها اگر ac>ab.(یعنی زاویه b بزرگتر از زاویه c است اگر و تنها اگر ضلع ac بزرگتر از ضلع ab باشد)
قضیه دو:در دو مثلث abc و xyz داریم و ab=xy و ac=xz .آنگاه a>x اگر و تنها اگر bc>yz .(این قضیه یک خاصیت شهودا بدیهی قیچی را بیان می کند!یعنی هرچه قیچی را بیشتر باز کنیم طول دهانه اش بیشتر می شود!)
مثلث abc را در نظر بگیرید که دو نیمساز bd و ce با هم برابرند.می خواهیم ثابت کنیم ab=ac
اثبات:
برهان خلف:فرض کنید ab با ac برابر نباشد،پس بدون کم شدن از کلیت مساله می توان فرض کرد ac>ab.
پس طبق قضیه یک داریم b>c. بنابر این b/2>c/2 پس در دو مثلث bdc و ceb طبق قضیه دو داریم cd>be.
حال از نقطه e پاره خط ef را موازی و برابر bd رسم کنید.پس چهارضلعی befd متوازی الاضلاع است.و مثلث efc متساوی الساقین است.حال دقت کنید که زاویه efd برابر b/2 است و زاویه ecd برابر c/2 است،پس با توجه به اینکه دو زاویه efc وecf برابرند، بدست می اید که زاویه dfc از زاویه dcf کوچکتر است.پس طبق قضیه یک داریم cd<df . اما be=df و قبلان داشتیم cd>be که تناقض است.

Maxwell_1989
14-12-2008, 23:16
ببخشيد من درست راه حلتونو متوجه نشدم. مي شه يه كم بيشتر توضيح بدين؟
سینوس پی هفتم رو با سینوس سه پی هفتم یکجا کنید و تبدیل به ضرب کنید و ادامه بدید.

panizir
14-12-2008, 23:23
سینوس پی هفتم رو با سینوس سه پی هفتم یکجا کنید و تبدیل به ضرب کنید و ادامه بدید.

من اين راهو رفتم فايده نداره.

ali_hp
15-12-2008, 01:12
ببخشيد من درست راه حلتونو متوجه نشدم. مي شه يه كم بيشتر توضيح بدين؟
سلام
راه حل ویرایش شد،دوباره بخونیدش!اگه بازم جاییش مبهم بود و متوجه نشدید بگید که اون قسمتو بیشتر توضیح بدم.

ali_hp
15-12-2008, 01:21
سینوس پی هفتم رو با سینوس سه پی هفتم یکجا کنید و تبدیل به ضرب کنید و ادامه بدید.
سلام
من متوجه نشدم منظورتونو....تبدیل به جمع یا ضرب؟با توجه به رادیکال هفتی که اونور تساوی داریم فکر نمی کنم با تبدیل به جمع کردن و ادامه دادن بشه به جواب رسید...خیلی جالبه اگه این مساله اینجوری هم حل بشه...!

Maxwell_1989
15-12-2008, 07:54
ببخشید.منظورم تبدیل به جمع بود.

1731
15-12-2008, 11:37
پست اشتباهی
چرا در ویرایش پست امکان پاک کردن پست نیست؟
به هر حال این پست اشتباهی است

panizir
15-12-2008, 14:32
سلام
دقت کنید که 0 و 2p/7 و 4p/7 و6p/7 و 8p/7 و 10p/7 و 12p/7 ریشه های معادله sin7x=0 هستند،همچنین:

[ برای مشاهده لینک ، لطفا با نام کاربری خود وارد شوید یا ثبت نام کنید ]

بنابر این sin(2p/7),sin(4p/7),sin(6p/7),sin(8p/7),sin(10p/7),sin(12p/7),sin0 ریشه های معادله زیر هستند:

[ برای مشاهده لینک ، لطفا با نام کاربری خود وارد شوید یا ثبت نام کنید ]

اما

[ برای مشاهده لینک ، لطفا با نام کاربری خود وارد شوید یا ثبت نام کنید ]

پس sin(2p/7),sin(4p/7),sin(6p/7),sin(8p/7),sin(10p/7),sin(12p/7)l ریشه های معادله زیر هستند:

[ برای مشاهده لینک ، لطفا با نام کاربری خود وارد شوید یا ثبت نام کنید ]

طبق قضیه ای می دانیم حاصلضرب ریشه های یک چند جمله ای برابر است با جمله ثابت ان ، تقسیم بر ضریب جمله با بزرگترین درجه،از طرفی:

[ برای مشاهده لینک ، لطفا با نام کاربری خود وارد شوید یا ثبت نام کنید ]

پس داریم:

[ برای مشاهده لینک ، لطفا با نام کاربری خود وارد شوید یا ثبت نام کنید ]




بسيار متشكرم.
ببخشيد من نمي دونستم چه جوري مي شه از يه پست تشكر كرد مجبور شدم يه پست ديگه بزنم.

panizir
15-12-2008, 20:34
دو تا سوال مثلثات ديگه:
ثابت كنيد:


Sin x + Sin (3x) + Sin (5x) + Sin (7x) = 4 Cosx . Cos(2x) + Cos (4x)

تبديل به حاصلضرب كنيد: (همه جملات بايد به ضرب (يا تقسيم) تبديل بشن و هيچ علامت جمعي نبايد بمونه. مشكل من هم همين شرطه)


2 + tan x + cot (2x)

ali_hp
16-12-2008, 15:30
دو تا سوال مثلثات ديگه:
ثابت كنيد:


Sin x + Sin (3x) + Sin (5x) + Sin (7x) = 4 Cosx . Cos(2x) + Cos (4x)

تبديل به حاصلضرب كنيد: (همه جملات بايد به ضرب (يا تقسيم) تبديل بشن و هيچ علامت جمعي نبايد بمونه. مشكل من هم همين شرطه)


2 + tan x + cot (2x)

فکر می کنم رابطه اول غلطه،قرار دهید x=0 ،بدست میاد 5=0
برای مساله دوم هم دقت کنید که
tanp+tanq=sin(p+q)/cospcosq
حالا به جای cot(2x+2)l بذارید tan(pi/2-2x-2)l و از رابطه بالا استفاده کنید.البته من فکر می کنم صورت این سوال هم باید یه چیز دیگه باشه!

panizir
21-12-2008, 17:30
صورت سوال دوم اينه:

tan(x)+ cot (2x) + 2

ali_hp
22-12-2008, 00:04
صورت سوال دوم اينه:

tan(x)+ cot (2x) + 2
سلام
[ برای مشاهده لینک ، لطفا با نام کاربری خود وارد شوید یا ثبت نام کنید ]

Hulk 2
22-12-2008, 23:03
سلام
من یه مسئله راحت دارم که در مورد درصد و از این چیزاست.ببینم حلش میکنین (خودم بلد نیستم ها).ممنون
>> صابر 2/5 (دوپنجم) از 1/2(یک دوم) پول خود را خرج کرده است . چند درصد از از پولش را خرج کرده است؟

MaFia_King
23-12-2008, 13:26
با سلام..

دوستان استاد رياضي عمومي1 ما حدودا 40تا تمرين داده بود براي حل كردنشون،من تونستم بعضي از مسايل رو حل كنم.ولي در بعضي از مسايل مشكل داشتم ،

اينجا مطرح مي كنم اگر امكان داره راهنمايي كنيد تا مشكلم حل شه.:11:

||ســـــپــــاس||[ برای مشاهده لینک ، لطفا با نام کاربری خود وارد شوید یا ثبت نام کنید ]

[ برای مشاهده لینک ، لطفا با نام کاربری خود وارد شوید یا ثبت نام کنید ]

[ برای مشاهده لینک ، لطفا با نام کاربری خود وارد شوید یا ثبت نام کنید ]

[ برای مشاهده لینک ، لطفا با نام کاربری خود وارد شوید یا ثبت نام کنید ]

[ برای مشاهده لینک ، لطفا با نام کاربری خود وارد شوید یا ثبت نام کنید ]

AR^2
27-12-2008, 23:02
بسمه تعالی
با سلام خدمت همه ی دوستان
من یه سوال دارم که جوابش برام خیلی مهمه!لطفاً اگه بلدید سریعاً پاسخ دهید.

اثباتش برام مهمه..
"مساحت چندضلعی های منتظم محاطی و محیطی"


با تشـــــــــــــــــکر

saeed-d
28-12-2008, 19:01
سلام
من نمیدونم این سوال رو پرسیدن یا نه ولی من میپرسم .
اثبات فرمول هرون رو به طور کامل میخواستم.

فرمول:

[ برای مشاهده لینک ، لطفا با نام کاربری خود وارد شوید یا ثبت نام کنید ]

AR^2
28-12-2008, 23:10
کسی نبود جواب بده!!!!


"مساحت چندضلعی های منتظم محاطی و محیطی"
با اثبات.....

saeed-d
29-12-2008, 18:51
سلام
من نمیدونم این سوال رو پرسیدن یا نه ولی من میپرسم .
اثبات فرمول هرون رو به طور کامل میخواستم.

فرمول:

[ برای مشاهده لینک ، لطفا با نام کاربری خود وارد شوید یا ثبت نام کنید ]

یادم رفت بگم
a+b+c/2 = p نصف محیط=p

کاپیتان رضائی
29-12-2008, 22:08
اقا قربونتون برم یکی به ما بگه که توی تابع ایکس در جزء صحیح ایکس چرا تو 1 حد نداره؟
ممنان

ali_hp
29-12-2008, 23:12
اقا قربونتون برم یکی به ما بگه که توی تابع ایکس در جزء صحیح ایکس چرا تو 1 حد نداره؟
ممنان
سلام
حد چپش میشه صفر و حد راستش میشه یک،پس حد چپ و راست برابر نیستن و تابع حد ندارد.

ali_hp
29-12-2008, 23:42
کسی نبود جواب بده!!!!


"مساحت چندضلعی های منتظم محاطی و محیطی"
با اثبات.....

سلام
مساحت n ضلعی منتظم محاطی در دایره ای به شعاع R:
فرض کنید AB یک ضلع n ضلعی باشد و H وسط این ضلع و O مرکز دایره باشد مساحت n ضلعی n برابر مساحت مثلث OAB است.و مساحت مثلث OAB برابر 1/2OA*OB*sin(AOB)=1/2R^2sin(360/n)l
پس داریم S=1/2nR^2sin(360/n)l
مساحت n ضلعی منتظم محیط بر دایره ای به شعاع R:
فرض کنید AB یک ضلع n ضلعی باشد و H وسط این ضلع باشد و O مرکز دایره باشد.مساحت n ضلعی n برابر مساحت مثلث OAB است.و مساحت مثلث OAB برابر 1/2OH*AB است.از طرفی داریم:AB=2Rtan(180/n)l
پس داریمS=nR^2tan(180/n)l

ali_hp
29-12-2008, 23:51
سلام
من نمیدونم این سوال رو پرسیدن یا نه ولی من میپرسم .
اثبات فرمول هرون رو به طور کامل میخواستم.

فرمول:

[ برای مشاهده لینک ، لطفا با نام کاربری خود وارد شوید یا ثبت نام کنید ]
سلام
با رابطه S=1/2bcsin(A)l شروع کنید،به توان دو برسانید،بجای sin^2(A)l بگذارید

1-cos^2(A)
حالا از قضیه کسینوسها استفاده کنید،یعنی به جای cosA عبارت

(b^2+c^2-a^2)/2bc قرار دهید.
حال یک رابطه داریم که در یک طرف ان S^2 است و در طرف دیگر ان عبارتی بر حسب a,b,c .که به سادگی می توان ان را ساده کرد و ثابت کرد که با عبارت

P(p-a)(p-b)(p-c)
برابر است.

dampayi
30-12-2008, 11:05
سلام
حد چپش میشه صفر و حد راستش میشه یک،پس حد چپ و راست برابر نیستن و تابع حد ندارد.
یا می شد همون اول بگیم چون پیوسته نیست (شکلشو همه بلدیم دیگه)!

dampayi
30-12-2008, 11:28
علی جان منم یه سوال دارم:
حاصل :

∑ (2^(k+1) + 3^(k) + 1)/5^(k-1)
مشکل من حساب کردنش نیست !!
می خواستم بدونم چرا جواب ها یکی نمی شه!!!
راه اول :
جمله اول می شه 8
جمله دوم می شه هیجده پنجم
یعنی قدر نسبت 9 تقسیم بر 20
پس جوابش می شه 8 تقسیم بر یک منهای نه بیستم
می شه 160 تقسیم بر 11 تقریبا 14.54
راه دوم:
جدا کنیم یعنی زیگما 2 به توان k ضرب در 2 کلش تقسیم بر 5 به توان k ضرب در 5 به توان منفی یک
برای سه تا جمله همینو بنویسم آخرش می شه
دو پنجم به توان کا ضربدر 10 به علاوه سه پنجم به توان کا ضربدر 5 به علاوه یک پنجم به توان کا ضربدر 5
که وقتی سیگما رو پخش کنیم و دونه دونه برای هر کدوم فرمولو بنویسیم می شه بیست سوم + پانزده دوم + پنج چهارم که می شه 185 دوازدهم تقریبا 15.41
چرا؟

AR^2
30-12-2008, 14:20
سلام
مساحت n ضلعی منتظم محاطی در دایره ای به شعاع R:
فرض کنید AB یک ضلع n ضلعی باشد و H وسط این ضلع و O مرکز دایره باشد مساحت n ضلعی n برابر مساحت مثلث OAB است.و مساحت مثلث OAB برابر 1/2OA*OB*sin(AOB)=1/2R^2sin(360/n)l
پس داریم S=1/2nR^2sin(360/n)l
مساحت n ضلعی منتظم محیط بر دایره ای به شعاع R:
فرض کنید AB یک ضلع n ضلعی باشد و H وسط این ضلع باشد و O مرکز دایره باشد.مساحت n ضلعی n برابر مساحت مثلث OAB است.و مساحت مثلث OAB برابر 1/2OH*AB است.از طرفی داریم:AB=2Rtan(180/n)l
پس داریمS=nR^2tan(180/n)l

با سلام مجدد و تشکر ویژه از دوست خوبمون ali_hp

راستش از این روش یکی از دوستام اثبات کرده.میخواستم ببینم روشی غیر از این وجود داره؟

اگه هست لطف کنید اونم بنویسید.

با تشکر ویژه از همگی

dampayi
30-12-2008, 14:48
ای وای چرا این عبارتو اینجوری نشون می ده!!!
به خدا من درست نوشتم!!!
علی جان برات می خونم!!!:
سیگما یک تابینهایت، دو به توان کای به علاوه یک ، به علاوه 3 به توان کا، به علاوه 1، کلش تقسیم بر 5 به توان کای منهای یک
ممنون

dampayi
31-12-2008, 12:10
آقا خیلی ممنون من خودم فهمیدم چرا غلط می شد!!
به خاطر اینکه اون ضابطه اولیه تصاعد هندسی نبود (جمع چندتا تصاعد هندسی بود)!!!!!!!!!!

pedramakaay
31-12-2008, 12:41
سلام...

می خواستم بدونم آیا کسی می دونه روش سه قطری در حل دستگاه معادلات خطی (ax=b) چی هست؟

از کجا می تونم اطلاعاتی در مورد این روش و الگوریتم کار کردن اون بدست بیارم؟

انواع روشهاي عددي حل مسائل مطرح در اكثر علوم مهندسي ، از جمله مهندسي سازه ، منجر به تشكيل دستگاه معادلات خطي مي شود. روشهاي تفاضلات محدود ،‌المان محدود، سختي، نرمي و ... در حل سازه ها منجر به تشكيل دستگاه معادلات خطي مي شود. يكي از روشهاي حل دستگاه معادلات خطي روشهاي تكراري مي باشد. روش تكراري شناخته شده و مرسوم در تحليل سازه ها روش پخش لنگر مي باشد. در اين روش كه دستگاه معادلات حاصل از روش شيب افت با روشهاي تكراري حل مي شود، منطبق بر روش گاوس سايدل مي باشد. در اين پايان نامه با بررسي هايي در روش پخش ممان نشان داده شده است كه جملات حاصل از روش پخش ممان در هر مرحله از عمليات پخش ، جملاتي از يك سري هندسي همگرا مي باشند. .و مجموع بينهايت تكرار كه به جواب دقيق معادلات ميل مي كند با يك عبارت صريح قابل بيان است. با كاربرد مفهوم هندسي بودن جملات حاصل از روش تكراري پخش ممان در حل تيرهاي سراسري كه ماتريس ضرائب دستگاه معادلات آن سه قطري است، مي توان حل دستگاه معادلات سه قطري را به صورت نهايي قالب بندي كرد. اين موضوع كه حل دستگاه معادلات سه قطري در حالت كلي را مي توان توسط فرمول بسته اي بدست آورد به نظر مي رسد از نظر رياضي حائز اهميت باشد.

هر کس می دونه لطفا راهنمایی کنه و منابعی رو در این زمینه اعلام کنه...با تشکر فراوان:11:

کاپیتان رضائی
31-12-2008, 19:38
سلام...

می خواستم بدونم آیا کسی می دونه روش سه قطری در حل دستگاه معادلات خطی (ax=b) چی هست؟

از کجا می تونم اطلاعاتی در مورد این روش و الگوریتم کار کردن اون بدست بیارم؟

انواع روشهاي عددي حل مسائل مطرح در اكثر علوم مهندسي ، از جمله مهندسي سازه ، منجر به تشكيل دستگاه معادلات خطي مي شود. روشهاي تفاضلات محدود ،‌المان محدود، سختي، نرمي و ... در حل سازه ها منجر به تشكيل دستگاه معادلات خطي مي شود. يكي از روشهاي حل دستگاه معادلات خطي روشهاي تكراري مي باشد. روش تكراري شناخته شده و مرسوم در تحليل سازه ها روش پخش لنگر مي باشد. در اين روش كه دستگاه معادلات حاصل از روش شيب افت با روشهاي تكراري حل مي شود، منطبق بر روش گاوس سايدل مي باشد. در اين پايان نامه با بررسي هايي در روش پخش ممان نشان داده شده است كه جملات حاصل از روش پخش ممان در هر مرحله از عمليات پخش ، جملاتي از يك سري هندسي همگرا مي باشند. .و مجموع بينهايت تكرار كه به جواب دقيق معادلات ميل مي كند با يك عبارت صريح قابل بيان است. با كاربرد مفهوم هندسي بودن جملات حاصل از روش تكراري پخش ممان در حل تيرهاي سراسري كه ماتريس ضرائب دستگاه معادلات آن سه قطري است، مي توان حل دستگاه معادلات سه قطري را به صورت نهايي قالب بندي كرد. اين موضوع كه حل دستگاه معادلات سه قطري در حالت كلي را مي توان توسط فرمول بسته اي بدست آورد به نظر مي رسد از نظر رياضي حائز اهميت باشد.

هر کس می دونه لطفا راهنمایی کنه و منابعی رو در این زمینه اعلام کنه...با تشکر فراوان:11:
هلاک شدم !! ببین قربونت من دوساعت گشتم رسیدم به یه وب که بهتره خودت بری اونجا پیداش کنی
باشه؟
آفرین قربونت


برای مشاهده محتوا ، لطفا وارد شوید یا ثبت نام کنید

AR^2
01-01-2009, 14:16
با سلام مجدد و تشکر ویژه از دوست خوبمون ali_hp

راستش از این روش یکی از دوستام اثبات کرده.میخواستم ببینم روشی غیر از این وجود داره؟

اگه هست لطف کنید اونم بنویسید.

با تشکر ویژه از همگی



دوستان کمک!

993
04-01-2009, 12:16
سلام
سوالم مربور به احتمالات میشه! اسونه ولی خودم یادم رفته!
7 تا کیسه داریم که توی هر کدوم 3 مهره ی رنگی وجود داره! حالا به چند مدل میشه این مهره ها رو خارج کرد؟
مثلا :
1-ابی-2-ابی-3قرمز-4سفید5-قرمز6-ابی7-قرمز
یه مدلشه! چجوری حساب کنم که چند مدل میشه؟

کاپیتان رضائی
04-01-2009, 12:23
سلام
سوالم مربور به احتمالات میشه! اسونه ولی خودم یادم رفته!
7 تا کیسه داریم که توی هر کدوم 3 مهره ی رنگی وجود داره! حالا به چند مدل میشه این مهره ها رو خارج کرد؟
مثلا :
1-ابی-2-ابی-3قرمز-4سفید5-قرمز6-ابی7-قرمز
یه مدلشه! چجوری حساب کنم که چند مدل میشه؟
کپی همین سوال تو جبر احتمال سوم دبیرستان موجوده که برای پیدا کردنش به گام به گام مراجعه شود
ویا دوستان جواب بدن

993
04-01-2009, 12:31
کپی همین سوال تو جبر احتمال سوم دبیرستان موجوده که برای پیدا کردنش به گام به گام مراجعه شود
ویا دوستان جواب بدن
متاسفانه من سالهاس که سوم دبیرستان رو تموم کردم و دیگه اون کتاب رو ندارم!:d حالا خودتون بلد نیستید که کمک کنید؟

ali_hp
04-01-2009, 12:34
سلام
سوالم مربور به احتمالات میشه! اسونه ولی خودم یادم رفته!
7 تا کیسه داریم که توی هر کدوم 3 مهره ی رنگی وجود داره! حالا به چند مدل میشه این مهره ها رو خارج کرد؟
مثلا :
1-ابی-2-ابی-3قرمز-4سفید5-قرمز6-ابی7-قرمز
یه مدلشه! چجوری حساب کنم که چند مدل میشه؟
اگر در هر کیسه از هر رنگ دقیقا یک مهره وجود داشته باشه،برای انتخاب هر مهره از هر کیسه سه حالت داریم،پس طبق اصل ضرب جواب میشه سه به توان هفت.

AR^2
04-01-2009, 14:18
دوستان کمک!



جایزه جایزه



50ساعت اینترنت شبانه(1-9)


فقط سریع باشید و اثبات دقیق ودرست باشه
2تا راه حل دیگه می خوام.
به ازای هر راه حل 50 ساعت
یعنی 2راه حل 100ساعت!!!!!1

MIG
05-01-2009, 23:21
دوستان كسي مي تونه اين سوال رو حل كنه با برهان خلف خيلي واجبه برام
فرض كنيد x1,x2,…,xn بردار هاي ويژه A ومقادير ويژه متناظر با λ 1, λ 2,…, λ n باشد. دراين صورت x1,x2,…xn مستقل خطي اند.يعني هيچكدام تركيب يا مجموع ويا ضريبي از ديگري نيست.

ali_exme
06-01-2009, 20:01
بوسیله انتگرال سه گانه حجم بیضی گون زی را حساب کنید.(2 ها همه توان)
x2/a2+y2/b2+z2/c2=1

panizir
07-01-2009, 20:03
من چند تا سوال دارم. اينا دوتاشه:
ابعاد پرحجم ترين مكعب مستطيل را كه بتوان آن را داخل يك بيضي گون محاط كرد را بيابيد
مطلوب است تعيين نسبت جعبه سربازي به شكل مكعب مستطيل و با حجم معين در صورتي كه بخواهيم مقدار جنس مورد نياز براي ساخت آن حداقل باشد.

pedramakaay
07-01-2009, 20:22
هلاک شدم !! ببین قربونت من دوساعت گشتم رسیدم به یه وب که بهتره خودت بری اونجا پیداش کنی
باشه؟
آفرین قربونت


برای مشاهده محتوا ، لطفا وارد شوید یا ثبت نام کنید

مرسی دوست عزیز...

سرچ کردم اما هیچی نبود....این صفحه که اصلا مربوط به بحث دیگریست:41:

هیچ کس نمی تونه منو راهنمایی کنه؟:41:

shitan
07-01-2009, 21:49
اثبات مبهم نبودن صفر به توان بینهایت

کاپیتان رضائی
07-01-2009, 22:46
سرچ کردم اما هیچی نبود....این صفحه که اصلا مربوط به بحث دیگریست

هیچ کس نمی تونه منو راهنمایی کنه؟
ای بابا ...
اون فهرستشو میگشتی حتما پیدا میکردی.از هر موضوعی زده بودن!

Hulk 2
07-01-2009, 23:00
[ برای مشاهده لینک ، لطفا با نام کاربری خود وارد شوید یا ثبت نام کنید ]
من رفتم همه چیز داشت در مورد ریاضی هم بود

m_belbasi
09-01-2009, 11:13
سلام آقاي مفيدي
من دانش دوم دبيرستان مدرسه شهيد بهشتي زنجان هستم مي خواستم بگم اگه ميشه لطف كنيد چند نمونه سوال از سوال هاي خودتون را كه سخت باشد براي امتحان دي كه شنبه داريم برام معرفي كنيد.
ممنون.

saber57
09-01-2009, 14:08
همه ريشه هاي حقيقي معادله درجه 4 زير را دقيقا تا 4 رقم اعشار بيابيد:
(x^4)-(2n+1)(x^2)-x-(n^2)+n-1=0
n=10^10
:5::31: با استفاده از نرم افزار matlab :

x1=-1.5538*10^5 , x2= -6.4359*10^4 i

x3=1.5538*10^5 , x2= 6.4359*10^4 i

جوابها دقیق و بدون خطا هستند (دارای دو ریشه حقیقی و دو موهوم)

AR^2
10-01-2009, 13:55
جایزه جایزه



50ساعت اینترنت شبانه(1-9)



فقط سریع باشید و اثبات دقیق ودرست باشه




2تا راه حل دیگه می خوام.



به ازای هر راه حل 50 ساعت



یعنی 2راه حل 100ساعت!!!!!1





با سلام خدمت دوستان

مهلتش تموم شد!!!!

دیگه بهش فکر نکنید.....

Parnyan
12-01-2009, 18:47
سلام

میشه یکی این سوال برام حل کنه ؟؟؟

[ برای مشاهده لینک ، لطفا با نام کاربری خود وارد شوید یا ثبت نام کنید ]

panizir
12-01-2009, 20:46
[ برای مشاهده لینک ، لطفا با نام کاربری خود وارد شوید یا ثبت نام کنید ]

دامنه gof ميشه x به طوري كه x عضو دامنه f و (f(x عضو دامنه g.

دامنه f ميشه x<2 اجتماع x>=3 و دامنه g مي شه x >= -3 پس (f(x بايد از 3- بزرگتر باشه.ميدونيم(f(x
عددي بين 1 و 1- است ( چون 1 به روي عدد صحيحي است) پس همواره از 3- بزرگتر است پس دامنه قسمت دوم هم همان دامنه (f(x مي شود در نتيجه دامنه gof همان دامنه (f(x يعني x<2 , x>=3 مي شود.

panizir
15-01-2009, 14:00
دو تا سوال ديگه هم من داشتم:


تابع( f(x,y با تغيير متغير x=e^s , y=e^t به تابع (g(s,t تبديل مي شود ثابت كنيد:

Gss+Gtt=x^2(fss)+y^2(fyy)+xfx+yfy

نشان دهيد كه در مبدا مشتق سويي f(x,y,z)=(x^2+y^2+z^2)^0/5 در هر سوي برابر واحد است ولي در مبدا گراديان ندارد

Vahid67
17-01-2009, 21:06
سلام
بجه ها ببخشید یه سوال ساده دارم آخه بلد بودم الان هرجی فکر می کنم یادم نمیاد!!!!
2 به توان n تعداد مینترم ها یا ماکسترم ها
در تشکیل مینترم ها و ماکسترم ها یه مرحله هست که باید اندیس i رو به عدد باینری NPT تبدیل کنیم میشه به من یادآوری کنید چجوری بود!!!

مثلاً : M0 عدد باینریش میشه 000
M3 عدد باینریش میشه 011
Mn-1 عدد باینریش میشه 111

Parnyan
19-01-2009, 23:13
سلام

اگه میشه جوابه این سوالارو یکی برام بزاره (plllllllllllz).....تا فردا میخوام :41:امتحان (جبر)دارم


>>>
[ برای مشاهده لینک ، لطفا با نام کاربری خود وارد شوید یا ثبت نام کنید ]


ممنون میشم ....

Parnyan
20-01-2009, 13:00
یکی لطف کنه این ُحل کنه plllllllllllzz

[ برای مشاهده لینک ، لطفا با نام کاربری خود وارد شوید یا ثبت نام کنید ]

panizir
20-01-2009, 14:29
[ برای مشاهده لینک ، لطفا با نام کاربری خود وارد شوید یا ثبت نام کنید ]
[ برای مشاهده لینک ، لطفا با نام کاربری خود وارد شوید یا ثبت نام کنید ]

ممنون میشم ....

براي سوال اول فرض خلف مي گيريم كه گوياست. به توان مي رسونيم. 5 مي بريم اون طرف سمت راست يه كسره كه مساويه يه عدد گويا ساده تر قرار مي ديم. حالا ثابت مي كنيم راديكال 6 گنگه. بازم فرض خلف مي گيريم به توان دو مي رسونيم طرفين وسطين مي كنيمدر مياد صورت كسر بايد بر 6 بخشپذير باشه ( چون توان 2ش بخشپديره خودشم هست) قرار مي ديم 6k به توان 2 مي رسونيم به دست مياد مخرج هم بر 6 بخشپذيره كه اول كه فرض كرديم صورت و مخرج نسبت به هم اولند تناقض مي شه.

سوال دوم:از راه استقرا ؛براي پايه n=1 كه درسته فرض مي كنيم براي n هم درست باشه هر دو طرفو در 1+راديكال 2 ضرب مي كنيم سمت چپ توان n+1 در مياد سمت راستو ساده مي كنيم به جواب مي رسيم.

سوال سوم: براي n=1 كه درسته براي n فرض مي كنيم درست باشه به هر دو طرف 2 ^(n+1) اضافه مي كنيم. طرف دومو ساده مي كنيم به جواب مي رسيم.

سوال سوم: 5 مجموعه به اين شكل انتخاب مي كنيم 1و9 ؛ 2و8 ؛ 3و7 ؛ 4و6 ؛ 5 . شش انتخاب داريم و 5 مجموعه. طبق اصل لانه كبوتري حداقل از يك مجموعه دو انتخاب داريم كه از هر مجموعه اي كه باشد مجموعشان 10 مي شود.

سوال چهارم:A U B زير مجموعه Aاشتراك B كه اون زير مجموعه A و B است.در نتيجه AUB زير مجموعه A و B است پس A زير مجموعه B و B زير مجموعه A است كه نتيجه مي شود با هم برابرند.

موفق باشيد.....

حامدرضائی
21-01-2009, 21:47
با تشکر از دوستان گرامی
خب بریم به سراغ مشتق.
ببخشید ما چرا برای پیدا کردن شیب یک خط مشتق میگیریم؟
آخه چه دلیلی داره که مشتق بگیریم؟
ممنون

Parser
22-01-2009, 06:53
ابعاد پرحجم ترين مكعب مستطيل را كه بتوان آن را داخل يك بيضي گون محاط كرد را بيابيد


به نام معشوق ازلي
سلام

فكر مي كنم هر يك از ابعاد مستطيل برابر يكي از شعاع هاي بيضي گون ضرب در دو، تقسيم بر راديكال سه باشند.

به عبارتي اگر ابعاد مكعب مستطيل و بيضي گون به ترتيب عبارت باشند از: a,b,c و r,s,t
داريم : 2/((1/2)^(r/a=s/b=t/c=((3
rst*8)/((27)^(1/2))=V)

Parser
22-01-2009, 06:58
[ برای مشاهده لینک ، لطفا با نام کاربری خود وارد شوید یا ثبت نام کنید ]

به نام معشوق ازلي
سلام

مي دانيم كه مجموعه ي A∩B زير مجموعه ي A است و A زير مجموعه ي AUB.
از آنجا كه طبق فرض داشتيم A∩B = AUB پس AUB زير مجموعه ي A∩B است.
نتيجه مي شود كه AUB زير مجموعه ي A است.

چون A زير مجموعه ي AUB و با توجه به فرض مي توان دريافت كه AUB زير مجموعه ي A است، پس A=AUB
به صورت مشابه براي مجموعه ي B نيز داريم B=AUB
بنابراين مي توان نوشت:

AUB = A∩B ===> A = B = AuB = A∩B

tara332
22-01-2009, 13:13
سلام.
یه سوال هندسه که راستش خودم فکر می کنم قشنگه:
در مثلث متساوی الساقینی به راس 20 درجه و ساق a و قاعده ی b ثابت کنید:
اندازه ی a بین 2b و 3b است!
از سایتتون هم ممنون

pari123
22-01-2009, 17:01
با تشکر از دوستان گرامی
خب بریم به سراغ مشتق.
ببخشید ما چرا برای پیدا کردن شیب یک خط مشتق میگیریم؟
آخه چه دلیلی داره که مشتق بگیریم؟
ممنون

خب معنی مشتق رو میدونین؟
مشتق خط مماس بر هر نقطه است.
و خط مماس همون شیب میشه.

راه دیگه ای واسه پیدا کردن شیب داری؟

chessmathter
22-01-2009, 21:55
سلام.
یه سوال هندسه که راستش خودم فکر می کنم قشنگه:
در مثلث متساوی الساقینی به راس 20 درجه و ساق a و قاعده ی b ثابت کنید:
اندازه ی a بین 2b و 3b است!
از سایتتون هم ممنون
aکه مقدارش برابراست
[ برای مشاهده لینک ، لطفا با نام کاربری خود وارد شوید یا ثبت نام کنید ]
اینم که بین 2bو3b است

Reman
23-01-2009, 13:41
با تشکر از دوستان گرامی
خب بریم به سراغ مشتق.
ببخشید ما چرا برای پیدا کردن شیب یک خط مشتق میگیریم؟
آخه چه دلیلی داره که مشتق بگیریم؟
ممنون

البته مشتق همون تعریف پیدا کردن شیب هست :
f(x2)-f(x1) / x2-x1
حالا در مشتق شما بجای x2 ، دلتا ایکس (مقداری کوچک ) + x1 رو قرار بده و دلتا ایکس رو به صفر میل بده که همون تعریف مشتق هست و در واقع شیب یک مقدار کوچک در روی نمودار رو پیدا می کنی.
:10:

eh_mn
28-01-2009, 10:23
سلام...

می خواستم بدونم آیا کسی می دونه روش سه قطری در حل دستگاه معادلات خطی (ax=b) چی هست؟

از کجا می تونم اطلاعاتی در مورد این روش و الگوریتم کار کردن اون بدست بیارم؟

انواع روشهاي عددي حل مسائل مطرح در اكثر علوم مهندسي ، از جمله مهندسي سازه ، منجر به تشكيل دستگاه معادلات خطي مي شود. روشهاي تفاضلات محدود ،‌المان محدود، سختي، نرمي و ... در حل سازه ها منجر به تشكيل دستگاه معادلات خطي مي شود. يكي از روشهاي حل دستگاه معادلات خطي روشهاي تكراري مي باشد. روش تكراري شناخته شده و مرسوم در تحليل سازه ها روش پخش لنگر مي باشد. در اين روش كه دستگاه معادلات حاصل از روش شيب افت با روشهاي تكراري حل مي شود، منطبق بر روش گاوس سايدل مي باشد. در اين پايان نامه با بررسي هايي در روش پخش ممان نشان داده شده است كه جملات حاصل از روش پخش ممان در هر مرحله از عمليات پخش ، جملاتي از يك سري هندسي همگرا مي باشند. .و مجموع بينهايت تكرار كه به جواب دقيق معادلات ميل مي كند با يك عبارت صريح قابل بيان است. با كاربرد مفهوم هندسي بودن جملات حاصل از روش تكراري پخش ممان در حل تيرهاي سراسري كه ماتريس ضرائب دستگاه معادلات آن سه قطري است، مي توان حل دستگاه معادلات سه قطري را به صورت نهايي قالب بندي كرد. اين موضوع كه حل دستگاه معادلات سه قطري در حالت كلي را مي توان توسط فرمول بسته اي بدست آورد به نظر مي رسد از نظر رياضي حائز اهميت باشد.

هر کس می دونه لطفا راهنمایی کنه و منابعی رو در این زمینه اعلام کنه...با تشکر فراوان:11:

سلام دوست عزیز.
برای حل دستگاه معادلات خطی بزرگ و تنک از روشهای تکراری استفاده میشه. که معروفترینشون روش گاوس-سایدل ، روش ژاکوبی و روش SOR هستن. یک منبع خیلی خوب رو می تونی از

برای مشاهده محتوا ، لطفا وارد شوید یا ثبت نام کنید
دانلود کنی.

کتاب Numerical Analysis نوشته J. Stoer و سایرین هم توضیحاتی در این زمینه داره.

Vict0Ria
10-02-2009, 11:28
با تشکر از دوستان گرامی
خب بریم به سراغ مشتق.
ببخشید ما چرا برای پیدا کردن شیب یک خط مشتق میگیریم؟
آخه چه دلیلی داره که مشتق بگیریم؟
ممنون

بزار من این مسئله رو قابل لمس تر توضیح بدم خوب شیب یعنی چی؟
چه موقع شیب یه خط افزایش میابد؟
شیب یعنی دلتای x به روی دلتای y
دو تا سرسره رو در نظر بگیر شیب کدوم بیشتره؟
خوب معلومه دیگه اونی که عرضش بیشتر از طولش باشه.
در نظر بگیر یه سرسره به صورت قائم وایستاده یعنی xش صفره در این صورت شیبش بینها یته.تا اینجا رو که قبول داری؟
خوب برای یک تابع داریم : f(x)=y پس:
دلتای y=دلتای f(x)
بقیش رو هم دوست خوبمون توضیح دادن:


البته مشتق همون تعریف پیدا کردن شیب هست :
f(x2)-f(x1) / x2-x1
حالا در مشتق شما بجای x2 ، دلتا ایکس (مقداری کوچک ) + x1 رو قرار بده و دلتا ایکس رو به صفر میل بده که همون تعریف مشتق هست و در واقع شیب یک مقدار کوچک در روی نمودار رو پیدا می کنی.

Arman_BM
11-02-2009, 11:11
بسم الله الرحمن الرحیم
دوستان من چند وقتی هست که سر این سوال گیر هستم.
برنامه نویسی بلد نیستید نترسید سوال اصلیم هیچ ربطی به برنامه نویسی نداره
for i =0 to 20 do
for j = 0 to i do
for k = 0 to j do
readln();

سوال اولیه این بود که دستور readln چند بار اجرا میشود.
خوب تعداد بار هایی که اجرا میشه برابر تعداد حالت هایی هست که میشه توی سه جایگاه کاملا مشابه اعداد صفر تا بیست رو نوشت. دقت کنین که جایگاه ها فرقی ندارن و مثلا 1-2-3 همون 3-2-1 هست.
.حالا برای حل این قسمت به ذهنم رسید که بیام معادله ی رو برو رو حساب کنم: x1+x2+x3<=60 با این شرط: xi<=20
سپس برای حذف تکرار جواب رو تقسیم بر 3! بکنم که تعداد جایگشت های سه جایگاه هست.
میخواستم ببینم این معادله چجوری حل میشه؟

x1+x2+x3<=60 با این شرط: xi<=20

movahedy
11-02-2009, 14:04
با عرض سلام و خسته نباشید

میخواستم از دوستان خواهش کنم که این مسئله ریاضی رو اثبات کنن و یک توضیح مختصر هم در مورد راه حلش ارائه نمایند.
مسئله رو در آدرس زیر قرار دادم :

برای مشاهده محتوا ، لطفا وارد شوید یا ثبت نام کنید
با تشکر

Iron
11-02-2009, 18:22
با عرض سلام و خسته نباشید

میخواستم از دوستان خواهش کنم که این مسئله ریاضی رو اثبات کنن و یک توضیح مختصر هم در مورد راه حلش ارائه نمایند.
مسئله رو در آدرس زیر قرار دادم :

برای مشاهده محتوا ، لطفا وارد شوید یا ثبت نام کنید
با تشکر

سلام

[ برای مشاهده لینک ، لطفا با نام کاربری خود وارد شوید یا ثبت نام کنید ]

اگر جاییش مبهم بود بگید تا توضیح بدم.

saber57
12-02-2009, 16:04
با تشکر از دوستان گرامی
خب بریم به سراغ مشتق.
ببخشید ما چرا برای پیدا کردن شیب یک خط مشتق میگیریم؟
آخه چه دلیلی داره که مشتق بگیریم؟
ممنون
معادله خط y=ax+b

شیب خط بین دو نقطه دلخواه A(x1,y1) , B(x2,y2 :
a = y2-y1/x2-x1=m=tan(teta)=delta y/delta x)= ax2+b-ax1-b/x2-x1
از طرفی dy/dx=a
بنابراین m=dy/dx=a

saber57
13-02-2009, 00:22
با سلام..

دوستان استاد رياضي عمومي1 ما حدودا 40تا تمرين داده بود براي حل كردنشون،من تونستم بعضي از مسايل رو حل كنم.ولي در بعضي از مسايل مشكل داشتم ،

اينجا مطرح مي كنم اگر امكان داره راهنمايي كنيد تا مشكلم حل شه.:11:

||ســـــپــــاس||[ برای مشاهده لینک ، لطفا با نام کاربری خود وارد شوید یا ثبت نام کنید ]













[ برای مشاهده لینک ، لطفا با نام کاربری خود وارد شوید یا ثبت نام کنید ]



[ برای مشاهده لینک ، لطفا با نام کاربری خود وارد شوید یا ثبت نام کنید ]


**********

[ برای مشاهده لینک ، لطفا با نام کاربری خود وارد شوید یا ثبت نام کنید ]


[ برای مشاهده لینک ، لطفا با نام کاربری خود وارد شوید یا ثبت نام کنید ]

****************


[ برای مشاهده لینک ، لطفا با نام کاربری خود وارد شوید یا ثبت نام کنید ]

************************
[ برای مشاهده لینک ، لطفا با نام کاربری خود وارد شوید یا ثبت نام کنید ]



*****************************
[ برای مشاهده لینک ، لطفا با نام کاربری خود وارد شوید یا ثبت نام کنید ]


:20:***************************************

[ برای مشاهده لینک ، لطفا با نام کاربری خود وارد شوید یا ثبت نام کنید ]


[ برای مشاهده لینک ، لطفا با نام کاربری خود وارد شوید یا ثبت نام کنید ]


************
[ برای مشاهده لینک ، لطفا با نام کاربری خود وارد شوید یا ثبت نام کنید ]




******************

[ برای مشاهده لینک ، لطفا با نام کاربری خود وارد شوید یا ثبت نام کنید ]

:46:

dampayi
13-02-2009, 16:20
سلام
آقا یه توضیح درباره اصل کمال به من می دید؟

saber57
13-02-2009, 20:34
در چه موردی هست اصل کمال؟

dampayi
13-02-2009, 23:06
در چه موردی هست اصل کمال؟

کرانداری مجموعه ها!

Arman_BM
14-02-2009, 02:37
بسم الله الرحمن الرحیم
دوستان من چند وقتی هست که سر این سوال گیر هستم.
برنامه نویسی بلد نیستید نترسید سوال اصلیم هیچ ربطی به برنامه نویسی نداره
for i =0 to 20 do
for j = 0 to i do
for k = 0 to j do
readln();

سوال اولیه این بود که دستور readln چند بار اجرا میشود.
خوب تعداد بار هایی که اجرا میشه برابر تعداد حالت هایی هست که میشه توی سه جایگاه کاملا مشابه اعداد صفر تا بیست رو نوشت. دقت کنین که جایگاه ها فرقی ندارن و مثلا 1-2-3 همون 3-2-1 هست.
.حالا برای حل این قسمت به ذهنم رسید که بیام معادله ی رو برو رو حساب کنم: X1+x2+x3<=60 با این شرط: Xi<=20
سپس برای حذف تکرار جواب رو تقسیم بر 3! بکنم که تعداد جایگشت های سه جایگاه هست.
میخواستم ببینم این معادله چجوری حل میشه؟

x1+x2+x3<=60 با این شرط: Xi<=20


سلام دوستان
یعنی سوال من انقدر سخت بود که هیچ کس حتی سعی هم نکرد حلش کنه؟!!!!
بابا سوال از کتاب گسسته ی گریمالدی هست.
لطفا دوستان اهل فن یک همتی بکنند.
ممنون.
یا حق

saber57
14-02-2009, 10:27
سلام
آقا یه توضیح درباره اصل کمال به من می دید؟
اصل موضوع کمال:
هر زیرمجموعه ناتهی A از اعداد حقیقی و از بالا کراندار دارای کوچکترین کران بالایی میباشد.کوچکترین کران بالایی را به sup A (سوپریمم)نمایش میدهند.کران پایینی و بزرگترین کران پایینی به طرق مشابه تعریف میشوند.از اصل موضوع کمال نتیجه میشود هر زیر مجموعه ناتهی B از اعداد حقیقی که از پایین کراندار باشد،دارای بزرگترین کران پایینی میباشد که آنرا با inf B (اینفیمم)نمایش میدهیم. به این لینک مراجعه کنید:

برای مشاهده محتوا ، لطفا وارد شوید یا ثبت نام کنید

اصل کمال: هر زیر مجموعه غیر تهی و از بالا کراندارباید دارای کوچکترین کران بالا باشد.
تعریف دیگر:
مجموعه‌ی R کامل است چون هر زیرمجموعه‌ی ناتمامی از R که از بالا کران‌دار باشد ، کوچک‌ترین کران بالای حقیقی دارد ، اما مجموعه‌ی Q کامل نیست ، چون زیرمجموعه‌هایی دارد که از بالا کران‌دار است ولی کوچک‌ترین کران بالای آن معلوم نیست . به این نکته « اصل کمال » می‌گویند .
تعریف سوپریمم(کوچک‌ترین کران بالای)و اینفیمم(بزرگترین کران پایینی):

سوپریمم و اینفیموم
سوپریمم و اینفیموم
فرض کنید S یک مجموعه جزئی مرتب و A زیر مجموعه ای از آن است. در این صورت:
تعریف کران بالا: هر عنصر M از S را یک کران بالای مجموعه A می گوییم اگر بعد از همه عناصر A باشد، یعنی به ازای هر عضو a از A داشته باشیم: a≤M.
تعریف سوپریمم : اگر یکی از کران های بالای A قبل از همه کران های بالای دیگر A باشد به آن سوپریمم یا کوچک ترین کران بالا می گوییم.
تعریف کران پایین: هر عنصر m از S را یک کران پایین مجموعه A می گوییم اگر قبل از همه عناصر A باشد، یعنی به ازای هر عضو a از A داشته باشیم: m≤a.
تعریف اینفیمم : اگر یکی از کران های پایین A بعد از همه کران های پایین دیگر A باشد به آن اینفیمم یا بزرگ ترین کران پایین می گوییم.

- سوپریمم و اینفیمم یک مجموعه مثل A را به ترتیب به صورت sup(A) و inf(A) نشان می دهیم.

saber57
14-02-2009, 17:09
سلام دوستان
یعنی سوال من انقدر سخت بود که هیچ کس حتی سعی هم نکرد حلش کنه؟!!!!
بابا سوال از کتاب گسسته ی گریمالدی هست.
لطفا دوستان اهل فن یک همتی بکنند.
ممنون.
یا حق
با سلام
من دقیقا منظور شما رو از عناصر تکراری نفهمیدم ولی برای x1+x2+x3<=5 با شرط xi<=3 جوابهای ممکن رو مینویسم شما عناصر تکراری رو برام بنویس تا سوال شما رو دقیقا متوجه بشم . اونوقت با نرم افزار Matlab برنامه این مساله رو برای شما مینویسم .
تعداد جوابهای ممکن:


[ برای مشاهده لینک ، لطفا با نام کاربری خود وارد شوید یا ثبت نام کنید ]

dampayi
14-02-2009, 17:13
مجموعه‌ی r کامل است چون هر زیرمجموعه‌ی ناتمامی از r که از بالا کران‌دار باشد ، کوچک‌ترین کران بالای حقیقی دارد

اما مجموعه‌ی q کامل نیست ، چون زیرمجموعه‌هایی دارد که از بالا کران‌دار است ولی کوچک‌ترین کران بالای آن معلوم نیست


!!!!!!!!!!!!!
مشکل مم همین جاست!!!
مگه q زیرمجموعه r نیست؟!!!!!!!!!

Arman_BM
14-02-2009, 19:27
من دقیقا منظور شما رو از عناصر تکراری نفهمیدم ولی برای x1+x2+x3<=5 با شرط xi<=3 جوابهای ممکن رو مینویسم شما عناصر تکراری رو برام بنویس تا سوال شما رو دقیقا متوجه بشم . اونوقت با نرم افزار matlab برنامه این مساله رو برای شما مینویسم

سلام سابر جان
بازم دم شما گرم
من نمیدونم بخش ترکیبیات انجمن اینهمه فعالیت داره، معمای هشت وزیر رو حل میکنن اونوقت جواب این سوال پیش و پا افتاده ی ما رو نمیدن!

توضیح راجب سوالم: توی سه جایگشت، اعداد یک تا بیست به چند حالت میتونن قرار بگین اگه تکرار وجود نداشته باشه. یعنی 1,3,5 - 1و5و3 - 3و5و1 و تمام شش حالتی که با این سه عدد میشه ساخت، فقط یک حالت حساب بشه. در حقیقت من سه تا جایگاه رو عینان مثل هم فرض کردم.
اما مطمئن نیستم که برای حذف تکرار باید تقسیم بر چی بکنم عبارت رو.

و میخوام از این راه حل کنم که با حل معادله ی x1+x2+x3=<60 با شرط x<=20
یعنی من در حقیقت دوتا سوال رو در یک سوال مطرح کردم.!!!!
اگه جواب این دو تارو جدا جدا هم بدین من راضیم!!!!

البته بگم من میخوام سوال رو از این راه حل کنم و دنبال جواب آخر از هر راهی نیستم.
چون نوشتن این تو کامپیوتر سادست اما من میخوام ا ترکیبیات، اونم از این روشش سوال رو حل کنم.
ممنون از شما.
بازم ممنون از شما!
یا حق

saber57
14-02-2009, 20:39
سلام سابر جان
بازم دم شما گرم
من نمیدونم بخش ترکیبیات انجمن اینهمه فعالیت داره، معمای هشت وزیر رو حل میکنن اونوقت جواب این سوال پیش و پا افتاده ی ما رو نمیدن!

توضیح راجب سوالم: توی سه جایگشت، اعداد یک تا بیست به چند حالت میتونن قرار بگین اگه تکرار وجود نداشته باشه. یعنی 1,3,5 - 1و5و3 - 3و5و1 و تمام شش حالتی که با این سه عدد میشه ساخت، فقط یک حالت حساب بشه. در حقیقت من سه تا جایگاه رو عینان مثل هم فرض کردم.
اما مطمئن نیستم که برای حذف تکرار باید تقسیم بر چی بکنم عبارت رو.

و میخوام از این راه حل کنم که با حل معادله ی x1+x2+x3=<60 با شرط x<=20
یعنی من در حقیقت دوتا سوال رو در یک سوال مطرح کردم.!!!!
اگه جواب این دو تارو جدا جدا هم بدین من راضیم!!!!

البته بگم من میخوام سوال رو از این راه حل کنم و دنبال جواب آخر از هر راهی نیستم.
چون نوشتن این تو کامپیوتر سادست اما من میخوام ا ترکیبیات، اونم از این روشش سوال رو حل کنم.
ممنون از شما.
بازم ممنون از شما!
یا حق
سلام آرمان
نیازی نیست که به عدد ثابتی تقسیم کنی باید همه حالتهای ممکن رو اول بنویسی بعد در حالتهای تکراری ، فقط یک حالت رو بنویسی . به من وقت بده جوابش رو در اولین فرصت برات قرار میدم. امیر :31:

Arman_BM
14-02-2009, 21:30
سلام آرمان
نیازی نیست که به عدد ثابتی تقسیم کنی باید همه حالتهای ممکن رو اول بنویسی بعد در حالتهای تکراری ، فقط یک حالت رو بنویسی . به من وقت بده جوابش رو در اولین فرصت برات قرار میدم. امیر :31:

سلام!
خوب اصلا میخواستی اسم آیدیت رو سابر نزاری که من هم ضایع نشم امیر جان!:40:
راستش من بیشتر دنبال روش حل معادلم تا جواب های خاص این سوال.
با برنامه نویسی میشه تمام زوج های مرتب رو درآورد. اما من نمیخوام اونجوری حلش کنم.
من میخوام حالت کلیش رو یاد بگیرم.
بازم ممنون از اینکه وقت گذاشتی رو سوال من.
در پناه یگانه امیر جهان!
یا حق!
:40:

chessmathter
14-02-2009, 21:49
سلام سابر جان
بازم دم شما گرم
من نمیدونم بخش ترکیبیات انجمن اینهمه فعالیت داره، معمای هشت وزیر رو حل میکنن اونوقت جواب این سوال پیش و پا افتاده ی ما رو نمیدن!

توضیح راجب سوالم: توی سه جایگشت، اعداد یک تا بیست به چند حالت میتونن قرار بگین اگه تکرار وجود نداشته باشه. یعنی 1,3,5 - 1و5و3 - 3و5و1 و تمام شش حالتی که با این سه عدد میشه ساخت، فقط یک حالت حساب بشه. در حقیقت من سه تا جایگاه رو عینان مثل هم فرض کردم.
اما مطمئن نیستم که برای حذف تکرار باید تقسیم بر چی بکنم عبارت رو.

و میخوام از این راه حل کنم که با حل معادله ی x1+x2+x3=<60 با شرط x<=20
یعنی من در حقیقت دوتا سوال رو در یک سوال مطرح کردم.!!!!
اگه جواب این دو تارو جدا جدا هم بدین من راضیم!!!!

البته بگم من میخوام سوال رو از این راه حل کنم و دنبال جواب آخر از هر راهی نیستم.
چون نوشتن این تو کامپیوتر سادست اما من میخوام ا ترکیبیات، اونم از این روشش سوال رو حل کنم.
ممنون از شما.
بازم ممنون از شما!
یا حق
ببخشید ما تورنمت داشتیم وقت نداشتیم به سوال شما جواب بدیم !!:angry:
یعنی چی سوال رو از این راه حل کنم تو سوال آسونو ورداشتی سخت ترش کردی اتفاقا جواب های اون نامعادله رو اینطوری حل میکنن
میخوام اعداد 1 تا 20تو 3 جایگشت بزاری وترتیب هم مهم نیست 3 حالت داریم
1)هر 3 تا متمایز 2)2 تا یکسان یکی متمایز3)هر سه یکسان
اولی که تعدادش برابر است با
[ برای مشاهده لینک ، لطفا با نام کاربری خود وارد شوید یا ثبت نام کنید ]
دومی برابر با 20*19
سومی برابر 20
پس تعداد جوابهای اون نامعادله برابر جمع این 3 تاست

Arman_BM
14-02-2009, 22:09
ببخشید ما تورنمت داشتیم وقت نداشتیم به سوال شما جواب بدیم !!:angry:
یعنی چی سوال رو از این راه حل کنم تو سوال آسونو ورداشتی سخت ترش کردی اتفاقا جواب های اون نامعادله رو اینطوری حل میکنن
میخوام اعداد 1 تا 20تو 3 جایگشت بزاری وترتیب هم مهم نیست 3 حالت داریم
1)هر 3 تا متمایز 2)2 تا یکسان یکی متمایز3)هر سه یکسان
اولی که تعدادش برابر است با
[ برای مشاهده لینک ، لطفا با نام کاربری خود وارد شوید یا ثبت نام کنید ]
دومی برابر با 20*19
سومی برابر 20
پس تعداد جوابهای اون نامعادله برابر جمع این 3 تاست

ممنون از کمکتون . راهنماییتون.
جوابتون رو میخونم و تحلیل میکنم.
ببخشید شما رو عصبانی کردم!

(دوستانه!)
یا حق

dampayi
14-02-2009, 22:44
دوستان منو درباره همون اصل کمال راهنمایی نمی کنن؟
مشکل من اینجاست که q زیر مجموعه r هست!

chessmathter
14-02-2009, 23:28
دوستان منو درباره همون اصل کمال راهنمایی نمی کنن؟
مشکل من اینجاست که q زیر مجموعه r هست!
تعریف اصل کمال در پیش دانشگاهی با اصل کمال در کتاب های مرجع متفاوت است !!! اینم از عجایبه دیگه!!!
خوب هست من چی بکنم!!!!:31:
مجموعه ای کامل است که هر زیر مجوعه ناتهی ازش و از بالا کران دار کرانش در همون مجموعه باشه (پیش دانشگاهی.!)
q کامل نیست چون مجوعه مثهA و کراندار از بالای میشه پیدا کرد
[ برای مشاهده لینک ، لطفا با نام کاربری خود وارد شوید یا ثبت نام کنید ]
که رادیکال 5 تو مجوعه گویا نیست
هر مجموعه نا تهی sاز اعداد حقیقی از بالا کران دار باشد سوپریمم دارد(کتابهای مرجع.!)

dampayi
14-02-2009, 23:50
خب مگه خودش نگفت r کامله!؟
پس هر زیر مجموعه از r که از بالا کراندار باشه دارای کوئچکترین کران بالاست!!!
مثلا همین مثالی که شما زدید مگه زیر مجموعهrنیست !!؟
پس چرا کرانداره و کوچکتیرین کران بابلا ندار]؟
مگه q زیر مجموعه r نیست؟!!!!!!

chessmathter
14-02-2009, 23:54
خب مگه خودش نگفت r کامله!؟
پس هر زیر مجموعه از r که از بالا کراندار باشه دارای کوئچکترین کران بالاست!!!
مثلا همین مثالی که شما زدید مگه زیر مجموعهrنیست !!؟
پس چرا کرانداره و کوچکتیرین کران بابلا ندار]؟
مگه q زیر مجموعه r نیست؟!!!!!!
کی گفته نداره.!!! کران داره ولی اگه مجوعه مرجع Qباشه کرانش در q نیست

saber57
15-02-2009, 09:59
!!!!!!!!!!!!!
مشکل مم همین جاست!!!
مگه q زیرمجموعه r نیست؟!!!!!!!!!
:20:این استدلال غلطه اگه فرض کنیم q یک زیر مجموعه باشه طبق اصل کمال باید اول یک کران بالا داشته باشد(فرض کردیم کل مجموعه اعداد گویا) اما میدونیم که مجموعه اعداد گویا پایانی ندارد (کران ندارد) پس این استدلال نادرسته . به همین دلیل ریاضیدانها اومدن q رو توی یک مجموعه اعداد حقیقی جا داده اند (اما پایان اون با پایان r فرق داره و هر دو مبهم هستند)

saber57
15-02-2009, 10:59
سلام!
خوب اصلا میخواستی اسم آیدیت رو سابر نزاری که من هم ضایع نشم امیر جان!:40:
راستش من بیشتر دنبال روش حل معادلم تا جواب های خاص این سوال.
با برنامه نویسی میشه تمام زوج های مرتب رو درآورد. اما من نمیخوام اونجوری حلش کنم.
من میخوام حالت کلیش رو یاد بگیرم.
بازم ممنون از اینکه وقت گذاشتی رو سوال من.
در پناه یگانه امیر جهان!
یا حق!
:40:
اگر دنبال تعداد جوابهای ممکن هستی در حالت کلی x1+x2+x3+...+xn<=a با شرط xi<=b
بطوری که a>=b*n جواب این هست :
در حالت کلی b^n جواب ممکن داریم و تعداد جوابهای ممکن هست:


[ برای مشاهده لینک ، لطفا با نام کاربری خود وارد شوید یا ثبت نام کنید ]

اگر دنبال الگوریتم برنامه نویسی اون هستی دونستن فرمول بالا کمکی به برنامه نمیکنه:20:

chessmathter
15-02-2009, 18:15
:20:این استدلال غلطه اگه فرض کنیم q یک زیر مجموعه باشه طبق اصل کمال باید اول یک کران بالا داشته باشد(فرض کردیم کل مجموعه اعداد گویا) اما میدونیم که مجموعه اعداد گویا پایانی ندارد (کران ندارد) پس این استدلال نادرسته . به همین دلیل ریاضیدانها اومدن q رو توی یک مجموعه اعداد حقیقی جا داده اند (اما پایان اون با پایان r فرق داره و هر دو مبهم هستند)
چه ربطی داره q که یک زیر مجموعه از r هست اصل کمال نمیگی که زیر مجموعه حتما کران بالا داره اصلا ربطی نداره زیر مجموعه یک مجوعه ممکن بیکران باشه پایانبعد مجموعه یعنی چی!!!!!!:31:
الان مشکل کجاست آقا اصل کمال میگه یه مجوعه کامل است اگر به ازای هر زیر مجموعه از بالا کران دار, کران بالای اون زیر مجموعه, در داخل همون مجوعه باشد و q کامل نیست به خاطر همون مجموعهََ َA که مثال زدم

chessmathter
15-02-2009, 18:23
اگر دنبال تعداد جوابهای ممکن هستی در حالت کلی x1+x2+x3+...+xn<=a با شرط xi<=b
بطوری که a>=b*n جواب این هست :
در حالت کلی b^n جواب ممکن داریم و تعداد جوابهای ممکن هست:


[ برای مشاهده لینک ، لطفا با نام کاربری خود وارد شوید یا ثبت نام کنید ]

اگر دنبال الگوریتم برنامه نویسی اون هستی دونستن فرمول بالا کمکی به برنامه نمیکنه:20:


این شرط قرمزه خیلی مهمه .:20::46:

Arman_BM
15-02-2009, 23:52
اگر دنبال تعداد جوابهای ممکن هستی در حالت کلی x1+x2+x3+...+xn<=a با شرط xi<=b
بطوری که a>=b*n جواب این هست :
در حالت کلی b^n جواب ممکن داریم و تعداد جوابهای ممکن هست:


[ برای مشاهده لینک ، لطفا با نام کاربری خود وارد شوید یا ثبت نام کنید ]

اگر دنبال الگوریتم برنامه نویسی اون هستی دونستن فرمول بالا کمکی به برنامه نمیکنه:20:








سلام

امیر عزیز دستت طلا، فورمول خیلی خوبی بهم دادی
آقا chessmathter از شما هم خیلی خیلی ممنون که هوای ما تازه راه ها رو دارید.
تازه کار نه! تازه راه!

امیر جان، نه برنامه نویسیش رو مشکلی ندارم،
موفق و پیروز باشی.
بازم ممنون
یا حق.

goldsnake
15-02-2009, 23:53
يه سوال در مورد برنامه matlab داشتم. تو اين برنامه چجوري ميشه نمودار هيستوگرام كشيد؟؟؟

dampayi
15-02-2009, 23:58
[ برای مشاهده لینک ، لطفا با نام کاربری خود وارد شوید یا ثبت نام کنید ](26).gifاین استدلال غلطه اگه فرض کنیم q یک زیر مجموعه باشه طبق اصل کمال باید اول یک کران بالا داشته باشد(فرض کردیم کل مجموعه اعداد گویا) اما میدونیم که مجموعه اعداد گویا پایانی ندارد (کران ندارد) پس این استدلال نادرسته . به همین دلیل ریاضیدانها اومدن q رو توی یک مجموعه اعداد حقیقی جا داده اند (اما پایان اون با پایان r فرق داره و هر دو مبهم هستند)
خیلی جالب !!
آخه:
از بچگیمون بهمون می گفتن q زیر مجموعه r است!!!

chessmathter جان یه سوال!!
r کامل هست یا نه؟

saber57
16-02-2009, 09:16
خیلی جالب !!
آخه:
از بچگیمون بهمون می گفتن q زیر مجموعه r است!!!

chessmathter جان یه سوال!!
r کامل هست یا نه؟
:rambo:اصل کمال مجموعه ای رو شامل میشه که از بالا یا پایین کراندار باشه یعنی در مجموعه اعداد گویا و یا حقیقی مجموعه ای رو که انتخاب میکنیم ، اول باید از بالا یا پایین کراندار باشه بعد برای اون سوپریمم یا اینفیمم پیدا کنیم . در مجموعه گویا اصل کمال برای هر زیرمجموعه صدق نمیکنه . ولی اگه بخواهیم صحت اصل کمال رو برای تمام مجموعه گویا بررسی کنیم با این مشکل مواجه هستیم که تمام مجموعه اعداد چون از بالا کراندار نیستند نمیشه بطور کلی این اصل رو براش صادق دونست . در مجموعه اعداد حقیقی و طبیعی اصل کمال برای زیر مجموعه های از بالا کراندار یا پایین همیشه درسته

saber57
16-02-2009, 10:02
يه سوال در مورد برنامه matlab داشتم. تو اين برنامه چجوري ميشه نمودار هيستوگرام كشيد؟؟؟
:rambo: با دستور ساده ( hist(y,x که x دامنه و y برد تابع هست اینم موضوعی که از help نرم افزار matlab پیدا کردم :


hist


Histogram plot
Syntax
n = hist(Y)
n = hist(Y,x)
n = hist(Y,nbins)
[n,xout] = hist(...)
hist(...)
hist(axes_handle,...)


Description


A histogram shows the distribution of data values.
n = hist(Y) bins the elements in vector Y into 10 equally spaced containers and returns the number of elements in each container as a row vector. If Y is an m-by-p matrix, hist treats the columns of Y as vectors and returns a 10-by-p matrix n. Each column of n contains the results for the corresponding column of Y.
n = hist(Y,x) where x is a vector, returns the distribution of Y among length(x) bins with centers specified by x. For example, if x is a 5-element vector, hist distributes the elements of Y into five bins centered on the x-axis at the elements in x. Note: use histc if it is more natural to specify bin edges instead of centers.
n = hist(Y,nbins) where nbins is a scalar, uses nbins number of bins.
[n,xout] = hist(...) returns vectors n and xout containing the frequency counts and the bin locations. You can use bar(xout,n) to plot the histogram.
hist(...) without output arguments produces a histogram plot of the output described above. hist distributes the bins along the x-axis between the minimum and maximum values of Y.
hist(axes_handle,...) plots into the axes with handle axes_handle instead of the current axes (gca).
Remarks


All elements in vector Y or in one column of matrix Y are grouped according to their numeric range. Each group is shown as one bin.
The histogram's x-axis reflects the range of values in Y. The histogram's y-axis shows the number of elements that fall within the groups; therefore, the y-axis ranges from 0 to the greatest number of elements deposited in any bin. The x-range of the leftmost and rightmost bins extends to include the entire data range in the case when the user-specified range does not cover the data range. If you want a plot in which this does not happen (that is, all bins have equal width), you can create a histogram-like display using the bar command.
The histogram is created with a patch graphics object. If you want to change the color of the graph, you can set patch properties. See the examples for more information. By default, the graph color is controlled by the current colormap, which maps the bin color to the first color in the colormap.

chessmathter
16-02-2009, 11:10
خیلی جالب !!
آخه:
از بچگیمون بهمون می گفتن q زیر مجموعه r است!!!

Chessmathter جان یه سوال!!
R کامل هست یا نه؟
بخدا q زیر مجوعه r هست بهتون درست گفتن .
R کامل هست q کامل نیست فکر نکنم کسی اصل کمال رو گرفته باشه ای خدا!!!!

chessmathter
16-02-2009, 11:18
:rambo:اصل کمال مجموعه ای رو شامل میشه که از بالا یا پایین کراندار باشه یعنی در مجموعه اعداد گویا و یا حقیقی مجموعه ای رو که انتخاب میکنیم ، اول باید از بالا یا پایین کراندار باشه بعد برای اون سوپریمم یا اینفیمم پیدا کنیم . در مجموعه گویا اصل کمال برای هر زیرمجموعه صدق نمیکنه . ولی اگه بخواهیم صحت اصل کمال رو برای تمام مجموعه گویا بررسی کنیم با این مشکل مواجه هستیم که تمام مجموعه اعداد چون از بالا کراندار نیستند نمیشه بطور کلی این اصل رو براش صادق دونست . در مجموعه اعداد حقیقی و طبیعی اصل کمال برای زیر مجموعه های از بالا کراندار یا پایین همیشه درسته
یعنی چی اصل کمال مجموعه هایی شامل میشه که از بالا یا پاین کراندار باشه پس r وn کراندارند؟
بابا اصل کمال میگه اگه یه مجوعه مثل r هر زیر مجموعه ازش ودر نظر بگیریم واون زیر مجوعه از بالا کراندار باشه و کرانش در خود همون مجموعه باشه(r) کامل هست و r کامله

dampayi
16-02-2009, 15:47
بخدا q زیر مجوعه r هست بهتون درست گفتن .
R کامل هست q کامل نیست فکر نکنم کسی اصل کمال رو گرفته باشه ای خدا!!!!

مگه شما نمی گی r کامل هست؟!!
خب پس طبق تعریف :
هر زیر مجموعه از r که از بالا کراندار باشه کران بالا هم داره! درسته؟
مگه اون مجموعه ای که شما صفحه قبل مثال زدید(رادیکال 5) زیر مجموعه r نیست؟ (x هاش عضو q بود و q هم طبق گفته شما زیر مجموعه r هست پس تمام x های اون مجموعه عضو r هست پس زیر مجموعه r هست!!


ولی از بالا کرانداره ، کران بالا نداره!!!

خیلی سوال من گنگه؟؟؟

chessmathter
16-02-2009, 16:41
مگه شما نمی گی r کامل هست؟!!
خب پس طبق تعریف :
هر زیر مجموعه از r که از بالا کراندار باشه کران بالا هم داره! درسته؟
مگه اون مجموعه ای که شما صفحه قبل مثال زدید(رادیکال 5) زیر مجموعه r نیست؟ (x هاش عضو q بود و q هم طبق گفته شما زیر مجموعه r هست پس تمام x های اون مجموعه عضو r هست پس زیر مجموعه r هست!!


ولی از بالا کرانداره ، کران بالا نداره!!!

خیلی سوال من گنگه؟؟؟
نگاه این مجموعه رو داشته باش
[ برای مشاهده لینک ، لطفا با نام کاربری خود وارد شوید یا ثبت نام کنید ]
این مجوعه کراندار از بالاست و کران بالاش هم رادیکال 5 هست
از طرفی هم زیر مجوعه R هست و هم زیر مجموعه Q .
ولی کران بالاش یعنی رادیکال 5 در R هست و در Q نیست
واسه همین Q کامل نیست چون یک زیر مجوعه از Q پیداکردیم(A) که کران بالاش در Q نیست
ولی R کامله چون هر زیرمجوعه ازش در نظر بگیریم که کران دار باشه کرانش تو یه R هست مثه همین A.

dampayi
16-02-2009, 19:08
دوست من ممنون فکر کنم فهمیدم!!

راستی من پست قبلم رو اصلاح می کنم منظورم از کران بالا کوچکترین کران بالا بود!!!!

میدونید من چی فکر می کردم؟
من فکر می کردم کرانش باید توی خود زیر مجموعه باشه!!
نه اصلا اینم فکر نمکی کردم!!
نمی دونم چی فکر می کردم!!!!!!!!!!
در هر صورت الان که فکر می کنم فهمیدم!!
(چقدر من فکر میکنم!!!)


بازم ممنون

saber57
16-02-2009, 20:15
چه ربطی داره q که یک زیر مجموعه از r هست اصل کمال نمیگی که زیر مجموعه حتما کران بالا داره اصلا ربطی نداره زیر مجموعه یک مجوعه ممکن بیکران باشه پایانبعد مجموعه یعنی چی!!!!!!:31:
الان مشکل کجاست آقا اصل کمال میگه یه مجوعه کامل است اگر به ازای هر زیر مجموعه از بالا کران دار, کران بالای اون زیر مجموعه, در داخل همون مجوعه باشد و q کامل نیست به خاطر همون مجموعهََ َa که مثال زدم
:31:دقیقا من توی پست قبلی میخواستم اینو بگم. دوستمون میخواست ب اصل کمال رو برای مجموعه اعداد گویا و حقیقی تعمیم بده . به این نحو که چون مجموعه حقیقی کامل هست پس زیر مجموعه اون که اعداد گویا هست باید از اصل کمال تبعیت کنه . اینکه مجموعه گویا زیر مجموعه حقیقی هست ، شکی درش نیست اما من با قیاس مخالفم چون برای صدق کردن اصل کمال در یک مجموعه اولا باید این اصل برای تمام زیر مجموعه های اون صدق کنه که با توجه به خاصیت اعداد گویا این اصل برای تمام زیر مجموعه های شامل اعداد گویا صدق نمیکنه ثانیا اعداد گویا به عنوان یک زیر مجموعه کلی از اعداد حقیقی کران بالایی نداره که بخواهیم با این استدلال بگیم خب حالا چون حقیقی کامل هست پس گویا کامل هست !
من یه بار دیگه اصل کمال رو مینویسم و فرضی رو که دوستمون مطرح کردند میگم :

اصل کمال : هر مجموعه ناتهی از a و از بالا کراندار ، دارای کوچکترین کران بالایی میباشد .
حالا سئوال دوستمون :
اگر مجموعه a رو که زیر مجموعه ناتهی از اعداد حقیقی هست مجموعه اعداد گویا فرض کنیم ، پس اصل کمال باید در اون صدق کنه ؟
پاسخ : یکی از شرایط قضیه لحاظ نشده و اون کراندار بودن از بالاست . همونطور که میدونیم اعداد گویا کران بالا ندارند !!!!!

امدوارم قانع شده باشید

afshin50cent
18-02-2009, 00:07
سلام...............من یه سوال دارم میخواستم ببینم تهی یک رابطه است یا نه؟؟؟؟؟؟؟؟؟؟

saber57
18-02-2009, 14:49
سلام...............من یه سوال دارم میخواستم ببینم تهی یک رابطه است یا نه؟؟؟؟؟؟؟؟؟؟

تهی یک مجموعه است با ضفر عنصر. منظورتون از رابطه بودن اون چیه؟ (لطفا توضیح دهید)

amir_rahmani
19-02-2009, 14:59
بی خیال خودم حلش کردم اگه میشه مدیرا پست را پاک کنن.

afshin50cent
19-02-2009, 21:46
تهی یک مجموعه است با ضفر عنصر. منظورتون از رابطه بودن اون چیه؟ (لطفا توضیح دهید)
منظورم همینه من سال سوم دبیرستانم رشته ریاضی فیزیک...........تو کتاب جبر و احتمال درس مجموعه ها یه همچین سوالی داره ولی معلم جبر که معلم حسابانمون هم هست میگه تهی رابطه نیست ولی معلم هندسمون میگه هست................واقعا یه سوال شده برا ما:41::18:................تعریفشم اگه پیدا نکردید بگید که بزارم اگه پیدا نکردید:11:رابطه رو میگم..................:10:

saber57
21-02-2009, 01:00
منظورم همینه من سال سوم دبیرستانم رشته ریاضی فیزیک...........تو کتاب جبر و احتمال درس مجموعه ها یه همچین سوالی داره ولی معلم جبر که معلم حسابانمون هم هست میگه تهی رابطه نیست ولی معلم هندسمون میگه هست................واقعا یه سوال شده برا ما:41::18:................تعریفشم اگه پیدا نکردید بگید که بزارم اگه پیدا نکردید:11:رابطه رو میگم..................:10:
امیدوارم جوابم درست باشه
:46:

تعریف رابطه :
هر زیر مجموعه ی A × B مانند R ، را یک رابطه از A به B می نامیم .

تعریف مجموعه تهی(از دانشنامه آزاد ویکیپدیا):
مجموعه تهی

از ویکی‌پدیا، دانشنامهٔ آزاد

مجموعهٔ تهی (Empty set) مجموعه‌ای‌ست که هیچ عضوی ندارد، و معمولً با علامت [ برای مشاهده لینک ، لطفا با نام کاربری خود وارد شوید یا ثبت نام کنید ] نشان داده می‌شود. به صورت دقیقتر می‌توان نوشت:
[ برای مشاهده لینک ، لطفا با نام کاربری خود وارد شوید یا ثبت نام کنید ]


اینجا رو خوب دقت کنید
به ازای هر مجموعه A، تهی زیر مجموعه ای از A است:



∀A: ∅ ⊆ A

به ازای هر مجموعه A، اجتماع تهی و A برابر A است:


∀A: A ∪ ∅ = A

به ازای هر مجموعه A، اشتراک تهی و A برابر تهی است:


∀A: A ∩ ∅ = ∅

به ازای هر مجموعه A، حاصلضرب دکارتی تهی و A برابر تهی است:


∀A: A × ∅ = ∅

تنها زیرمجموعه مجموعه تهی، خودش است:


∀A: A ⊆ ∅ ⇒ A = ∅

تعداد اعضای مجموعه تهی صفر است و به عبارتی متناهی است:

|∅| = 0
حالا این دو تعریف رو کنار هم قرار میدم نتیجش با خودت


1-هر زیر مجموعه ی A × B مانند R ، را یک رابطه از A به B می نامیم .


2-به ازای هر مجموعه A، تهی زیر مجموعه ای از A است:

‎∀A: ∅ ⊆ A


نتیجه گیری : اگر دو مجموعه دلخواه A و B دارای زیر مجموعه R باشند ، به این علت که طبق تعریف 2 ،تهی زیر مجموعه ای از هر مجموعه و از جمله R هست . بنابراین یک رابطه(البته برای هر دو مجموعه دلخواه ) به حساب میاد.

k1kz
22-02-2009, 05:30
سلام به همگی من یه سوال دارم اگه کسی می تونه لطفا راهنمایی کنه:
چطوری میشه یه الگوریتمی طراحی کنیم که توابع دلخواه ما را برای مقادیر بزرگ تقریب کنه و الگوریتم در مرتبه زمانی لگاریتمی باشه؟ Logspace
یعنی فرض کنید میخواهیم تابع ((f(n)=Log(Bessel I0(n را برای مقادیر ...,n= 2^2 ,2^4 ,2^8 ,2^16
برآورد کنیم و زمان حل مساله را هم اندازه میگیریم این زمان برای nهای داده شده باید تناسب هندسی داشته باشه یعنی زمان حل مساله در هر n نسبت به n قبلی در یک عدد ثابت ضرب شده باشه.من اینکار رو برای هر تابعی که خواستم با برنامه Mathematica انجام دادم ولی نمیدونم چطوری برنامه اینکارو میکنه؟ جالب اینجاست که روشهای مرسوم تقریب توابع مثل سریهای تیلور و فوریه و چبیشف و کسرهای پده (pade) و کسرهای متوالی هیچکدوم در زمان لگاریتمی نمیتونن تقریب کنن من امتحان کردم.
اگه کسی چنین الگوریتمی سراغ داره لطفا بگه یا لینک کنه
ممنون
کیوان

k1kz
22-02-2009, 06:08
سلام کسی کتاب یا مقاله خوب برای حل معادلات بازگشتی غیر خطی سراغ داره؟
مثلا اینو چطوری دقیق حل کنیم؟ a(n+1)=(a(n)^2+1),a0=0

saber57
22-02-2009, 06:09
چطوری میشه یه الگوریتمی طراحی کنیم که توابع دلخواه ما را برای مقادیر بزرگ تقریب کنه و الگوریتم در مرتبه زمانی لگاریتمی باشه؟ logspace
کیوان

اگه ممکنه بیشتر توضیح بدید

k1kz
22-02-2009, 06:27
اگه ممکنه بیشتر توضیح بدید


یک مقاله خیلی کلی و ساده در این رابطه اینجاست ببینید:
[ برای مشاهده لینک ، لطفا با نام کاربری خود وارد شوید یا ثبت نام کنید ]

k1kz
22-02-2009, 07:19
با سلام

دوستان به لینک زیر سری بزنید و نرم افزار geometer's sketchpad را که مخصوص تولید تصاویر متحرک ریاضی است، دانلود کنید. از این نرم افزار می توان برای آموزش مفاهیم ریاضی در سطوح مختلف استفاده کرد.

[ برای مشاهده لینک ، لطفا با نام کاربری خود وارد شوید یا ثبت نام کنید ] ([ برای مشاهده لینک ، لطفا با نام کاربری خود وارد شوید یا ثبت نام کنید ] .com%2fsketchpad%2fgspdemos.html)

دانلود نرم افزار با حجم2.17 مگابایت:

[ برای مشاهده لینک ، لطفا با نام کاربری خود وارد شوید یا ثبت نام کنید ]


ممنون هستم جالب بود.:40:

k1kz
22-02-2009, 07:40
اگه ممکنه بیشتر توضیح بدید

نظریه‌ی پیچیدگی محاسباتی شاخه‌ای از علوم کامپیوتر و ریاضی است که به بررسی دشواری حل مسائل به وسیله‌ی رایانه (به عبارت دقیق‌تر به‌ صورت الگوریتمی) می‌پردازد. این نظری بخشی از نظریه‌ی محاسباتی است که با منابع مورد نیاز برای حل یک مساله سروکار دارد. عمومی‌ترین منابع زمان (چقدر زمان برای حل کردن مساله لازم است) و فضا (چقدر حافظه مورد نیاز است) می‌باشند. سایر منابع می‌تواند تعداد پروسسور‌های موازی (در حالت پردازش موازی) و … باشند. اما در این مقاله ما در مورد عواملی مثل عوامل بالا بحثی نکرده‌ایم. باید به این نکته توجه داشت که نظریه پیچیدگی با نظریه قابل حل بودن متفاوت می‌باشد. این نظریه در مورد قابل حل بودن یک مساله بدون توجه به منابع مورد نیاز آن، بحث می‌کند. بعد از این نظریه که بیان می‌کند کدام مسائل قابل حل می‌باشند و کدام مسائل غیرقابل حل، این سوال به نظر طبیعی می‌رسد که درجه سختی مساله چقدر است. نظریه پیچیدگی محاسبات در این زمینه می‌باشد.
برای یک توضیح مختصر در این باره میتونید به سایت آکادمیست مراجعه کنید.

به نقل از سایت آکادمیست دات آی آر

saber57
24-02-2009, 14:33
نظریه‌ی پیچیدگی محاسباتی شاخه‌ای از علوم کامپیوتر و ریاضی است که به بررسی دشواری حل مسائل به وسیله‌ی رایانه (به عبارت دقیق‌تر به‌ صورت الگوریتمی) می‌پردازد. این نظری بخشی از نظریه‌ی محاسباتی است که با منابع مورد نیاز برای حل یک مساله سروکار دارد. عمومی‌ترین منابع زمان (چقدر زمان برای حل کردن مساله لازم است) و فضا (چقدر حافظه مورد نیاز است) می‌باشند. سایر منابع می‌تواند تعداد پروسسور‌های موازی (در حالت پردازش موازی) و … باشند. اما در این مقاله ما در مورد عواملی مثل عوامل بالا بحثی نکرده‌ایم. باید به این نکته توجه داشت که نظریه پیچیدگی با نظریه قابل حل بودن متفاوت می‌باشد. این نظریه در مورد قابل حل بودن یک مساله بدون توجه به منابع مورد نیاز آن، بحث می‌کند. بعد از این نظریه که بیان می‌کند کدام مسائل قابل حل می‌باشند و کدام مسائل غیرقابل حل، این سوال به نظر طبیعی می‌رسد که درجه سختی مساله چقدر است. نظریه پیچیدگی محاسبات در این زمینه می‌باشد.
برای یک توضیح مختصر در این باره میتونید به سایت آکادمیست مراجعه کنید.

به نقل از سایت آکادمیست دات آی آر
مساله شما در زمینه مهندسی کامپیوتر نرم افزار هست . فکر کنم دتبال پیدا کردن الگوریتمی برای حل مساله پیچیدگی میگردید. شاید این کتاب مفید باشه :


C++طراحي الگوريتم ها با شبه كدهاي

ريچارد نيپوليتان , كيومرث نعيمي پور
نویسندگان
عين الله جعفرنژاد قمي
مترجمین


لینک :

برای مشاهده محتوا ، لطفا وارد شوید یا ثبت نام کنید

k1kz
24-02-2009, 15:07
مساله شما در زمینه مهندسی کامپیوتر نرم افزار هست . فکر کنم دتبال پیدا کردن الگوریتمی برای حل مساله پیچیدگی میگردید. شاید این کتاب مفید باشه :


C++طراحي الگوريتم ها با شبه كدهاي


ريچارد نيپوليتان , كيومرث نعيمي پور

نویسندگان

عين الله جعفرنژاد قمي
مترجمین






لینک :

برای مشاهده محتوا ، لطفا وارد شوید یا ثبت نام کنید


نظریه پیچیدگی قصد دارد اثبات کند که کلا کلاس P=NP است یعنی هر الگوریتم غیر قطعی در زمان چند جمله ای را میتوان به صورت الگوریتم قطعی در زمان چند جمله ای نوشت .مثلا در سال 2002 الگوریتم AKS برای اولین بار نشان داد که پیدا کردن برای اعداد اول این مطلب درست است و اعداد اول در کلاس پیچیدگی P قرار دارند. اما سوال من در باره تقریب توابع بدیهی است که در کلاس P است یعنی در زمان چند جمله ای و قطعی از روش سریهای تیلور حل میشود.ولی من دنبال الگوریتمی در کلاس Log Space هستم نه P اینکه P=NP باشد یا نباشد این مشکل را حل نمیکند در واقع :
LogSpace زیرمجموعه یا مساوی P است.
و P زیرمجموعه یا مساوی NP است.

k1kz
24-02-2009, 15:30
یک توضیح ساده برای اینکه دقیقا بگم می خواهم چه کار کنم اینه:
فرض کن ما یک عدد بزرگ داریم n میخواهیم مقدار تابع دلخواه مثلا BesselJ را محاسبه کنیم اگر از سری های تیلور استفاده کنیم بعلت همگرایی کند این سری ها باید بیشتر از n جمله از سری های تیلور را محاسبه کنیم یعنی در برنامه نویسی آن باید یک حلقه n تایی داشته باشیم تا اینکه به دقت مورد نظر برسیم ولی این اصلا مطلوب نیست و خیلی طول میکشه ولی مثلا اگه با برنامه Mathematica بخواهیم اینکارو زمان حل این مساله برابر یک عدد ثابت در لگاریتم n میشه یعنی برنامه یک حلقه ای به طول Log n رو اجرا میکنه.البته باید بگم سریهای تیلور برای n های کوچک یعنی بین صفر و یک خیلی سریع همگرا میشه ولی برای اعداد بزرگ نه.

k1kz
24-02-2009, 16:17
آقا صابر از توجه تون ممنون هستم ولی فکر کنم متوجه سوالم نشدید
یه توضیح ساده برای اینکه دقیقا بگم چیکار میخواهم کنم اینه:
فرض میخواهیم تابع BesselI0 رو برای یک n بزرگ حساب کنیم اگه از سریهای تیلور استفاده کنیم مجبوریم بیشتر از n
جمله از سریها را حساب کنیم. بعلت همگرایی کند این سریها برای اعداد بزرگ اگه بخواهیم برای محاسبه این تابع برنامه بنویسیم باید یک حلقه به طول بیشتر از n داشته باشیم ولی اگه بخواهیم اینکارو با برنامه Mathematica انجام بدهیم میبینیم که زمان حل مساله توسط برنامه برابر یک عدد ثابت در Log n میشه یعنی برنامه از یک روش پیچیده استفاده میکنه که اینقدر سریع جواب میده .البته بدیهیست که سریهای تیلور برای اعداد کوچک بین صفر و یک خیلی سریع همگرا میشه و تا هر
دقتی میشه سریع محاسبه کرد ولی برای اعداد بزرگ نه . این یک مساله در آنالیز عددی پیشرفته است اگه کتاب یا مقاله در باره آنالیز عددی پیشرفته که این مطلب رو توضیح داده باشه سراغ دارین لطفا بگین

aquaquest
24-02-2009, 17:07
سلام
دوستان گرامی لطف کنید معادله زیر را با توضیح کامل برایم حل کنید

[ برای مشاهده لینک ، لطفا با نام کاربری خود وارد شوید یا ثبت نام کنید ]

با تشکر از شما عزیزان

saber57
24-02-2009, 20:14
سلام
دوستان گرامی لطف کنید معادله زیر را با توضیح کامل برایم حل کنید

[ برای مشاهده لینک ، لطفا با نام کاربری خود وارد شوید یا ثبت نام کنید ]

با تشکر از شما عزیزان

نیازی به بسط معادلات نداری


[ برای مشاهده لینک ، لطفا با نام کاربری خود وارد شوید یا ثبت نام کنید ]

در محدود مورد نظر پاسخ x=4 ,y=9 (دو عدد مربع کامل 2 و 3 هستند)

k1kz
24-02-2009, 22:22
مساله شما در زمینه مهندسی کامپیوتر نرم افزار هست . فکر کنم دتبال پیدا کردن الگوریتمی برای حل مساله پیچیدگی میگردید. شاید این کتاب مفید باشه :


C++طراحي الگوريتم ها با شبه كدهاي

ريچارد نيپوليتان , كيومرث نعيمي پور

نویسندگان
عين الله جعفرنژاد قمي
مترجمین




لینک :

برای مشاهده محتوا ، لطفا وارد شوید یا ثبت نام کنید


یک مثال ساده عددی:احتمالا شما تابع انتگرال لگاریتمی را میشناسید.این تابع به این صورت تعریف میشود
[ برای مشاهده لینک ، لطفا با نام کاربری خود وارد شوید یا ثبت نام کنید ]
اگر بخواهیم این تابع را برای یک عدد بزرگ مثلا [ برای مشاهده لینک ، لطفا با نام کاربری خود وارد شوید یا ثبت نام کنید ] محاسبه کنیم چقدر طول میکشد؟ اگر از سریهای تیلور استفاده کنیم خیلی خیلی زمان طولانی باید منتظر بمانیم شاید حتی چند سال ولی برنامه Mathematica این کار را در کمتر از یک ثانیه انجام میدهد.نگاه کنید:

[ برای مشاهده لینک ، لطفا با نام کاربری خود وارد شوید یا ثبت نام کنید ]
من هیچ روشی به ذهنم نرسید که اینقدر سریع جواب بده.این مبحث مربوط به آنالیز عددی پیشرفته است .احتمالا توی ارشد ریاضی کاربردی تدریس میشه

saber57
25-02-2009, 10:47
در واقع شما میخواهید الگوریتمی که برنامه برای حل ارائه میده رو بدونید.برای همین مثالی که زدید ،به الگوریتمهای بکاررفته نیاز دارید.همون چیزی که احتمالا در ماشین حسابهای مهندسی هم استفاده میشه

k1kz
25-02-2009, 17:01
در واقع شما میخواهید الگوریتمی که برنامه برای حل ارائه میده رو بدونید.برای همین مثالی که زدید ،به الگوریتمهای بکاررفته نیاز دارید.همون چیزی که احتمالا در ماشین حسابهای مهندسی هم استفاده میشه


درسته البته در مورد ماشین حساب که بیشتر توابع مقدماتی داره و اعداد ورودی خیلی بزرگ نداره به نظر خیلی سخت نمیرسه مثلا توابع مثلثاتی همه متناوب هستند و تابع نمایی و توابع هیپربولیک هم برای مقادیر کوچک همون سریهای تیلور جواب میده وبرای مقادیر بزرگ هم که تو ماشین حساب جا نمیشه و ماشین حساب پیغام خطا میده و بنابراین فکر نمیکنم خیلی سخت باشه ولی کامپیوتر فرق داره.
من بخاطر همین سوال وچند تا سوال دیگه تصمیم گرفتم برم ارشد ریاضی کاربردی بخونم خدا کنه قبول بشم
راستی من یه سایتی هم پیدا کردم که سرفصل دروس ارشد و داره ببینید:

[ برای مشاهده لینک ، لطفا با نام کاربری خود وارد شوید یا ثبت نام کنید ]

k1kz
27-02-2009, 23:31
سلام بر همگی
از دوستان چه کسی به تحقیقات تئوریک علاقمنده ؟ تا به حال شده سعی کنید یه فرمول و یا یک روش جدید برای یک حل مساله خاص توی ریاضی بدست بیارین؟ از بازی کردن با فرمولها لذت می برین؟ اصلا به نظر شما مهمترین مساله حل نشده توی ریاضی کدومه؟ لطفا نظر بدین.
ممنون

mahsa1469
28-02-2009, 18:30
سلام دوستان،من یه سوال داشتم که سخت نیست اما نمی دونم چه جوری باید حلش کنم تو رو خدا زود جواب بدین تا فردا لازم دارم:
انتگرال رادیکال تانژانت xدی x

chessmathter
28-02-2009, 20:54
سلام دوستان،من یه سوال داشتم که سخت نیست اما نمی دونم چه جوری باید حلش کنم تو رو خدا زود جواب بدین تا فردا لازم دارم:
انتگرال رادیکال تانژانت xدی x
یا من قاطی کردم سوال سخت گرفتم و راه آسونی داره یا تو...
من که 1 ساعت جون کندم پدرم در اومد.:wac: این چه بود.:31::20:
هر جا مشکلی هست بگین توضیح دهم اگه هم مشکل محاسباتی داره بگین درست کنم چون دیگه حال چک ندارم.:31:
[ برای مشاهده لینک ، لطفا با نام کاربری خود وارد شوید یا ثبت نام کنید ]

saber57
01-03-2009, 04:02
سلام دوستان،من یه سوال داشتم که سخت نیست اما نمی دونم چه جوری باید حلش کنم تو رو خدا زود جواب بدین تا فردا لازم دارم:
انتگرال رادیکال تانژانت xدی x
:rambo:اینم یک راه حل دیگه:


[ برای مشاهده لینک ، لطفا با نام کاربری خود وارد شوید یا ثبت نام کنید ]



تنها کاری که میکنیم اینه که صورت ومخرج کسر رو با عبارت 2u^2 جمع میکنیم و مخرج رو مربع کامل میکنیم و بعدش هم ساده هست . این فرمول رو از هندبوک انتگرال پیدا کردم . .میتونید اونو برای انتگرال بکار ببرید:


[ برای مشاهده لینک ، لطفا با نام کاربری خود وارد شوید یا ثبت نام کنید ]



با جایگذاری مقادیر:


[ برای مشاهده لینک ، لطفا با نام کاربری خود وارد شوید یا ثبت نام کنید ]

صفحه 176 جواب درست رو قرار دادم

chessmathter
01-03-2009, 09:52
:rambo:اینم یک راه حل دیگه:


[ برای مشاهده لینک ، لطفا با نام کاربری خود وارد شوید یا ثبت نام کنید ]



تنها کاری که میکنیم اینه که صورت ومخرج کسر رو با عبارت 2u^2 جمع میکنیم و مخرج رو مربع کامل میکنیم و بعدش هم ساده هست . این فرمول رو از هندبوک انتگرال پیدا کردم . .میتونید اونو برای انتگرال بکار ببرید:


[ برای مشاهده لینک ، لطفا با نام کاربری خود وارد شوید یا ثبت نام کنید ]



با جایگذاری مقادیر:


[ برای مشاهده لینک ، لطفا با نام کاربری خود وارد شوید یا ثبت نام کنید ]



از خط سوم راه حلت فک کنم مشکل داره.صورت مخرج کسرو دیده بودیم در عددی ضرب یا تقسیم کنیم یا چیزی کم و همونو زیاد بکنیم ولی اینکه بهشون یه عبارت اضافه کنیم و برابر بشه نه.!:31::20::46: